ANCC FNP review questions

Lakukan tugas rumah & ujian kamu dengan baik sekarang menggunakan Quizwiz!

A 3-week-old male presents because of a sudden onset of bilious vomiting of several hours' duration. He is irritable and refuses to breastfeed, but his stools have been normal. He was delivered at term after a normal pregnancy and has had no health problems to date. Physical examination shows a fussy child with a distended abdomen. Radiography of the abdomen shows a corkscrew appearance of the duodenum. Which of the following is the most likely diagnosis? A. Infantile colic B. Necrotizing enterocolitis C. Hypertrophic pyloric stenosis D. Midgut volvulus

A. Infantile colicB. Necrotizing enterocolitisC. Hypertrophic pyloric stenosis D. Midgut volvulus Incorrect Correct Answer: D. Midgut volvulus The most common type of intestinal rotation in pediatric patients is an incomplete rotation that predisposes to midgut volvulus. Volvulus may present in three ways: as a sudden onset of bilious vomiting and abdominal pain in a neonate; as a history of "feeding problems" with bilious vomiting that appears to be a bowel obstruction; or less commonly, as failure to thrive with severe feeding intolerance. Volvulus occurs because the narrow mesenteric base, which develops as a result of malrotation, allows the small bowel to twist around the superior mesenteric artery. This leads to vascular compromise of large portions of the midgut. classic finding on abdominal plain films is the corkscrew sign. However, plain film can be entirely normal. The upper gastrointestinal contrast study is considered the gold standard for diagnosing volvulus. Incorrect Answers: A. Infantile colic usually begins during the second week of life and typically occurs in the evening. It is characterized by screaming episodes and a distended or tight abdomen. Its etiology has yet to be determined. There are no abnormalities on physical examination and ancillary studies, and symptoms usually resolve spontaneously around 12 weeks of age. B. Necrotizing enterocolitis is typically seen in the distressed neonate in the intensive-care nursery, but it may occasionally be seen in the healthy neonate within the first two weeks of life. The child will appear ill, with symptoms including irritability, poor feeding, a distended abdomen, and bloody stools. Abdominal plain films will show pneumatosis intestinalis caused by gas in the intestinal wall, which is diagnostic of the condition. C. Hypertrophic pyloric stenosis is a narrowing of the pyloric canal caused by hypertrophy of the musculature. It usually presents during the third to fifth weeks of life. Projectile vomiting after feeding, weight loss, and dehydration are common. The vomitus is always nonbilious, because the obstruction is proximal to the duodenum. If a small olive-size mass cannot be felt in the right upper or middle quadrant, ultrasonography will confirm the diagnosis.

A 3-year-old female was seen by the nurse practitioner 3 days earlier and diagnosed with a cold. She is otherwise healthy and has no drug allergies. Today, she returns for evaluation of ear pain. She is afebrile. Otoscopic examination reveals a bulging red tympanic membrane with opacification. Which of the following treatments is most appropriate? A. Amoxicillin B. Levofloxacin C. Bactrim D. Azithromycin

Correct Answer: A. Amoxicillin Amoxicillin is the first-line treatment and gold standard for acute otitis media. In PCN-allergic patients, azithromycin, Bactrim, and levofloxacin can be prescribed. Second-line treatment criteria include a history of antibiotic use in the past 3 months, no response to amoxicillin, or a severe case of AOM, with high fever and pain. Second-line drugs include Amoxicillin/clavulanate (Augmentin), cefuroxime, or third-generation cephalosporins. Incorrect Answers: (B), (C), (D) In PCN-allergic patients, azithromycin, Bactrim, and levofloxacin can be prescribed.

A 10-month-old girl with no prior medical history presents to the emergency room with signs of bronchiolitis. The patient's parents report that she developed a cough 3 days ago, and today she developed rhinorrhea, wheezing, and a fever of 100.4°F. Physical examination reveals a temperature of 100.5°F, and wheezing is evident on pulmonary exam. Physical examination also reveals right tympanic membrane fullness, with loss of light reflex and erythema. The patient keeps pulling on her ear. It is December, and this is the 10th patient this month who has presented with the same symptoms. Which of the following is the most appropriate therapy for this patient? A. Amoxicillin with analgesics B. Bronchodilators C. Corticosteroids D. Ribavirin

Correct Answer: A. Amoxicillin with analgesics The patient's age, lower respiratory tract symptoms, and the spike in cases seen during this time of year indicate that the patient most likely has respiratory syncytial virus (RSV). RSV is a viral infection that attacks the lower respiratory tract, causing bronchiolitis or pneumonia. While RSV commonly affects young children, it can be seen in adults. While uncomplicated RSV is treated with supportive therapy, the patient also has evidence of otitis media. The 2013 AAP/AAFP guideline recommends either immediate treatment or observation (with pain control) for children between 6 and 24 months with unilateral nonsevere AOM and for children ≥24 months with unilateral or bilateral nonsevere AOM. Since this child has evidence of unilateral AOM, either observation or treatment with antibiotics are appropriate. Individually randomized trials that used stringent diagnostic criteria demonstrated that children under the age of two benefits from antibiotic therapy, including those with nonsevere unilateral AOM. Incorrect Answers: B. Bronchodilators are not recommended for the routine treatment of RSV bronchiolitis. While bronchodilators may provide modest short-term improvement, they do not improve long-term outcomes and may have adverse effects. Bronchodilators also increase the cost of care and should only be used if a single trial leads to a prompt favorable response. C. Corticosteroids are not recommended for the routine treatment of RSV bronchiolitis in infants. However, corticosteroids may be helpful in older children and adults who have RSV-associated bronchial reactivity, especially those who have a history of asthma that may have been exacerbated by the recent RSV infection. Corticosteroids can potentially decrease bronchial swelling and airway obstruction through their inhibition of the inflammatory cascade, but studies have shown that this effect is minimal.

A 23-year-old woman with no health insurance presents to the free community clinic with vaginal discharge. She reports a new sexual partner last week. She describes the discharge as thick. She is also experiencing dysuria. You do a wet mount which is negative for clue cells or signs of yeast. You believe she has chlamydia. What would be your recommended first-line treatment regimen for this patient? A. Ceftriaxone and azithromycin B. Azithromycin C. Levofloxacin D. Metronidazole

Correct Answer: A. Ceftriaxone and azithromycin The first-line treatment for chlamydia is azithromycin 1 gram as a single dose. However, given this patient is in the free clinic without available further testing, you should be concerned about gonorrhea co-infection as well and treat empirically with an intramuscular injection of ceftriaxone. Incorrect Answers: B. Azithromycin is the first-line treatment for chlamydia, but this patient warrants gonorrhea co-treatment. C. Fluoroquinolones can be used as an alternative therapy in patients who are allergic to macrolides or who demonstrate treatment resistance. D. Metronidazole is used to treat trichomoniasis and bacterial vaginosis, but not chlamydia.

A 34-year-old woman is following up with you for a complaint of significant breast pain for the last year. She has had normal physical exams and recently finished a mammogram and breast ultrasound without any abnormalities. At her last visit you advised supportive garments, which slightly improved her symptoms. What would be the next best step in treatment? A. Diclofenac B. Danazol C. Tamoxifen D. Bioidentical hormone cream

Correct Answer: A. Diclofenac NSAIDs such as diclofenac, as well as acetaminophen, are recommended as first-line agents for breast pain. Incorrect Answers: B. Danazol is an androgen which is FDA approved for the treatment of breast pain. It is typically reserved for severe or refractory symptoms. C. Tamoxifen, a selective estrogen receptor modulator is used as off-label treatment of severe breast pain which has failed other treatments. D. Compounded hormonal regimens are not recommended

A depressed 75-year-old woman with a history of myocardial infarction s/p stent, cardiac arrhythmia (on coumadin), and emphysema presents to your office for the continuation of care. Which medication would you choose to treat her depression? A. Escitalopram B. Nortriptyline C. Doxepin D. Selegiline

Correct Answer: A. Escitalopram SSRIs are the first-line treatment choice for depressive disorders in older patients. They are generally well- tolerated and safe. Treating depression in older patients requires a thorough review of comorbid illnesses and medications. There should also be careful follow-up and monitoring of side effects. Incorrect Answers: B. Tricyclic antidepressants (TCA) are generally advised against in patients with cardiac arrhythmias. They can also worsen or affect narrow angle glaucoma , constipation, and BPH. TCAs can cause confusion. C. Doxepin is also a TCA (see explanation for B) D. MAO inhibitors require special dietary and medication restrictions (many aged cheeses, meats, for example). This is not generally the first choice in medications for antidepressant medications. One positive of MAOIs is that MAOI medications have very little cardiac conduction effects.

A 12-year-old boy complains of abrupt onset of scrotal pain upon awakening this morning. He has had nausea and vomiting for the past hour. He is afebrile and his heart rate is 100/minute. His scrotum is red and swollen and the right testicle is higher than the left. When you stroke the inner thigh on the right, there is no movement of the testicle. Which of the following is an indicated treatment for this condition? A. Manual reduction B. Antibiotics C. Topical azole cream D. Chemotherapy

Correct Answer: A. Manual reduction The patient, in this case, has a presentation consistent with testicular torsion. Patients with testicular torsion are usually young men between the ages of 10 and 20. They will complain of sudden onset of scrotal pain, usually in the middle of the night or upon awakening. The scrotum will be red and swollen and the affected testicle is usually positioned higher than the unaffected testicle, often closer to the body. The cremasteric reflex refers to elevation of the testicle when the ipsilateral thigh is stroked. This reflex is absent in testicular torsion. Testicular torsion is a urologic emergency. Torsion interrupts the blood supply to the testicle and permanent damage ensues if not reduced within 6 hours. After 24 hours, the testicle will become gangrenous and must be surgically removed. The nurse practitioner should call 911 and the patient should undergo Doppler ultrasound in the ED. Manual detorsion can be accomplished in the ED by the "open book" maneuver, but a surgical reduction is necessary if a manual reduction is unsuccessful. Incorrect Answers: B. Antibiotics would be appropriate for the treatment of epididymitis.C. Topical azole creams are used to treat balanitis, which is candida infection of the glans penis. D. Chemotherapy is appropriate for testicular cancer, which usually presents in young male adults who complain of a sensation of heaviness or aching a nodule or testicular enlargement, or testicular tenderness.

Which of the following reflexes is elicited by allowing the infant's head to move back suddenly when lifted a few inches off the mattress, resulting in the abduction and upward movement of the arms followed by adduction and flexion? A. Moro reflex B. Root reflex C. Jones reflex D. Babinski reflex

Correct Answer: A. Moro reflex The Moro reflex is elicited in this manner. Incorrect Answers: B. The root reflex describes when babies turn their faces towards a stimulus and make sucking motions with their mouths when their cheeks or lips are touched. C. There is no common newborn reflex referred to as the "Jones reflex." D. The Babinski reflex describes a baby's big toe moving upward and other toes fanning out after the sole of the baby's foot has been firmly stroked.

A 32-year-old male presents to the office with a new diagnosis of cluster headaches. What is the mainstay of cluster headache abortive therapy? A. Sumatriptan B. Verapamil C. Prednisone D. Divalproex

Correct Answer: A. Sumatriptan Sumatriptan (as well as zolmitriptan) with supplemental oxygen is the mainstay of abortive therapy. Incorrect Answers: B. Verapamil is a first-line drug for prophylaxis and treatment of chronic cluster headaches and has the most evidence in the treatment of cluster headaches. Electrocardiographic monitoring should be performed due to potential cardiac effects. C. Prednisone is most useful as a bridging therapy until another prophylactic medication is established. Bridging therapy is a useful way to use the least amount of triptans as possible in patients with frequent attacks. D. Other prophylactic medications that are used but do not have as much evidence include divalproex, topiramate, and ergotamine.

A 21-year-old female presents to a clinic requesting oral contraceptives. She has recently engaged in sexual intercourse with her boyfriend. They are currently using condoms to prevent pregnancy. Her last period was one week ago. Which of the following conditions is not a contraindication for starting a combined estrogen-progestin oral contraceptive pill? A. History of deep vein thrombosis B. Migraine headache without aura C. Factor V Leiden mutation D. History of pulmonary embolism

Correct Answer: B. Migraine headache without aura Migraine without aura is not a contraindication for starting a combined estrogen progestin oral contraceptive pills. Adolescents who have migraine with aura or other neurological changes should not be started on combined estrogen progestin oral contraceptive pills. Incorrect Answers: A. History of deep vein thrombosis (DVT) is a contraindication for starting a combined estrogen progestin oral contraceptive pill. C. Factor V Leiden mutation is a contraindication for starting a combined estrogen progestin oral contraceptive pill. D. History of pulmonary embolism is a contraindication for starting a combined estrogen progestin oral contraceptive pill.

A 63-year-old woman presents to your clinic with a complaint of urinary leakage. She states for the past several months she feels she is constantly leaking small amounts of urine. She goes to the bathroom frequently but doesn't feel she is emptying her bladder. She feels her urine stream is weak and she is having nocturia. She denies any change with sneezing. She is not having significant urgency. What type of incontinence is this woman experiencing? A. Stress incontinence B. Overflow incontinence C. Urge incontinence D. Mixed incontinence

Correct Answer: B. Overflow incontinence Overflow incontinence is usually due to under activity of the detrusor muscle or bladder outlet obstruction. It typically presents with continuous leaking or dribbling of urine, incomplete bladder emptying, weak stream, frequency, and nocturia. Incorrect Answers: A. Stress incontinence presents with urine leakage which is involuntary and increases with intraabdominal pressure such as coughing, sneezing, or physical exertion. C. Urge incontinence describes a frequent or near-continuous feeling of needing to void immediately, with urine leakage either before or after urination. D. Mixed incontinence traditionally is used to describe a patient who is having symptoms of both stress and urge incontinence.

A 44-year-old women presents at your office with a request for assistance with smoking cessation. Which of the following has been shown to be the most effective pharmacologic therapy for cessation? A. Nicotine replacement therapy B. Varenicline C. Bupropion D. Nortriptyline

Correct Answer: B. Varenicline Varenicline blocks nicotine receptors from behind bound by nicotine, thus reducing the 'rewarding' effects of smoking. In head-to-head trials, it was found to be more effective than nicotine replacement or bupropion. Early concerns about psychiatric side effects have later been found to be unsupported. Incorrect Answers: A. Nicotine replacement therapy with patches, gum, or lozenges has been found to be effective for nicotine cessation as well, but less effective than varenicline. C. Bupropion is a norepinephrine and dopamine enhancing agent which is also effective for smoking cessation, especially in patients with depression. However, it is less effective than varenicline. D. Nortriptyline is a tricyclic antidepressant which is a second-line agent for smoking cessation.

A 28-year-old female presents to the office for contraceptive followup. The etonogestrel contraceptive implant should be removed and replaced how long after insertion? A. 1 year B. 2 years C. 3 years D. 5 years

Correct Answer: C. 3 years The etonogestrel contraceptive implant (also known as Nexplanon or Implanon) is a progestin loaded into a rod that is inserted subcutaneously under the skin of the arm and provides contraception for three years, then it needs to be removed and replaced with another implant or another method of contraception. Incorrect Answers: A. 1 year is too few unless a woman desires to have it removed to become pregnant or due to side effects, the implant would still be effective after one year. B. 2 years is also too few unless the reasons stated above. D. 5 years is beyond the life of the implant and it may not provide adequate contraception if left in place this long.

In an experiment, a mutation is introduced into the fungal genome targeting a gene that is involved in the synthesis of ergosterol. The mutant strain is cultured in media containing a common antifungal drug. Compared to the wild-type controls grown in the same media, the mutant strain had an increased growth rate. Which of the following agents was most likely contained in the media? A. Micafungin B. Caspofungin C. Amphotericin B D. None of the above

Correct Answer: C. Amphotericin B Amphotericin B belongs to a class of antifungal agents known as polyenes. These agents bind to sterols that make up vital components of the fungal cell wall, mainly ergosterol. This interaction decreases the fluidity of the membrane, leading to decreased membrane integrity and ultimately fungal cell death. Mutations in ergosterol molecules and replacement with similar sterol molecules are common mechanisms of amphotericin resistance, as seen in this case. A useful mnemonic about amphotericin B: AmphoTEARicin TEARS holes in fungal membrane (binds ergosterol and forms pores) AmphoTERRible: AE: fever/chills, hypotension, nephrotox, arrhythmia, anemia, IV phlebitis Incorrect Answers: A and B. Caspofungin and micafungin, echinocandins, act through inhibition of glucan synthesis. A mutation in ergosterol would not impact their efficacy. D. Amphotericin B (a polyene) binds ergosterol in the fungal cell wall.

An 18 year old female presents to clinic for follow up after starting combined estrogen and progesterone oral contraceptive pills. Which of the following side effects is most likely due to the estrogen component of the pill? A. Abdominal bloating B. Mood changes C. Breast tenderness D. Acne

Correct Answer: C. Breast tenderness Breast tenderness is a side effect caused by the estrogen in oral contraceptive pills. Other side effects caused by estrogen may include irregular periods, nausea, headache, fluid retention, and hypertension. These side effects may be decreased by switching to an oral contraceptive pill with a lower estrogen dose. Incorrect Answers: A. Abdominal bloating is more likely to be caused by the progestin component of oral contraceptive pills. Other side effects caused by progestin included hirsutism and acne. Male-pattern baldness is a rare progestin side effect. B. Mood changes are more likely to be caused by the progestin component of oral contraceptive pills. D. Acne is more likely to be caused by the progestin component of oral contraceptive pills. This adrenergic effect is caused by the increased free testosterone circulating in the blood.

The mechanism by which thiazide and thiazide-like diuretics lower blood pressure is incompletely understood, but the decrease in blood pressure appears to be a response to initial volume loss. This fall in blood pressure is initially blunted by the action of the renin- angiotensin system, but the blood pressure will still remain low or continue to fall slowly over as long as 12 weeks. Which mechanism explains this continued blood pressure lowering after urinary sodium returns to normal? A. Parasympathetic activation B. Sympathetic inactivation C. Decreased peripheral vascular resistance D. Decreased cardiac contractility

Correct Answer: C. Decreased peripheral vascular resistance Immediately after the initiation of a thiazide diuretic, patients experience diuresis and reduction in blood pressure due to volume loss. This response is blunted by hypovolemia-induced activation of the renin- angiotensin system. Thiazides achieve their diuretic action via inhibition of the Na+/Cl− cotransporter in the renal distal convoluted tubule. The transporter facilitates the absorption of sodium from the distal tubules back to the interstitium and. By decreasing sodium reabsorption, thiazide use results in an increase in fluid loss to urine, which leads to decreased extracellular fluid (and plasma volume). This volume loss results in diminished venous return, increased renin release (upregulation), reduced cardiac output and decreased blood pressure Eventually diuresis decreases (within a few weeks) because the previously upregulated renin-angiotensin- aldosterone system returns to prior baseline. The plasma volume and cardiac output partially rise toward the baseline level, while the systemic vascular resistance falls. This secondary effect is more pronounced with long-acting thiazide diuretics. The mechanism for this vasodilatory effect is unclear. The maintenance of reduction in blood pressure is due to this small, secondary vasodilatory effect that is characterized by decreased peripheral vascular resistance. Overall, expect a drop of about 10 mmHg with thiazide diuretics. Incorrect Answers: A. Parasympathetic activation leads to decreased cardiac output through depression of heart rate and contractility. This is not the mechanism of continued blood pressure reduction after sodium homeostasis is achieved. B. Sympathetic inactivation is not the mechanism thought to maintain low blood pressure after the initial period of volume loss.

A 33 y/o patient requests a refill of her birth control pills. She heard from a friend that birth control pills protect a woman from cancer and would like additional information. You inform the patient that oral contraceptives can regulate menstrual cycles and reduce dysmenorrhea and you also provide which of the following information? A. Oral contraceptives can reduce the risk of ovarian cancer while taking the medication B. Oral contraceptives have a slight but negligible increase in endometrial cancer risk C. Oral contraceptives have an increased risk of cervical cancer D. Oral contraceptives have an increased risk of breast cancer

Correct Answer: C. Oral contraceptives have an increased risk of cervical cancer Oral contraceptives have an increased risk of cervical cancer especially in women with HPV, but this increased risk should be carefully explained to patients. Although slightly increased, the absolute risk is still quite low. The risk of invasive cervical cancer increases with increasing oral contraceptive use; however, the risk decreases after stopping and then returns to nonuser risk after 10 years. Incorrect Answers: A. Oral contraceptives are shown to reduce the risk of ovarian cancer for 30 years after stopping use. The reduced risk is found in women that are "ever users". Larger reductions in ovarian cancer risk are gained with longer use of oral contraceptives. Oral contraceptives are considered protective of ovarian cancer. There is some data to suggest the same effect with fallopian tubal cancer. At this time, there is no data extending this protection to contraceptive patches/rings. B. Oral contraceptives are shown to reduce the risk of endometrial cancer for 30 years after stopping use. D. The data on breast cancer has been conflicting. Some studies show no association and others show an increased use with current (but not past) use. The absolute risk is very low (one additional case per 7690 women per year), and the majority of studies have not demonstrated an association between oral contraceptive use and the risk of breast cancer later in life.

43-year-old patient with past medical history of mild asthma in his childhood presents for management of hypertension. He has heard that some antihypertensive medications can worsen breathing in asthmatic patients. Which of the following medications would be most likely to cause bronchospasm in a patient with asthma? A. Metoprolol B. Atenolol C. Propranolol D. Bisoprolol

Correct Answer: C. Propranolol Propranolol is the only beta-blocker listed with is non-selective. Beta-blockers can be cardioselective, or non- selective. Cardioselectivity refers to the ability to affect predominantly beta 1 receptors rather than beta 2 receptors. Beta 1 receptors are located mainly in the heart and mediate the sympathetic nervous system's direct effects on the heart. Beta 2 receptors are located predominantly in the peripheral vascular system and other organs like the lungs. Although all beta-blockers affect beta receptors, some affect subsets of receptors differently. This has significant consequences in terms of side effects, as beta-blockers are known to cause extra-cardiac symptoms such as worsened bronchospasms in asthmatics. Although beta-1 selective blockers are safer than nonselective beta blockers, and would be preferred in patients with asthma, they should still be used with caution in patients with asthma, particularly in those with severe obstruction or markedly reduced pulmonary function at baseline. Incorrect Answers: A. B. and D. These are all cardioselective beta-blockers.

A 26-year old female patient entering her third trimester of pregnancy has routinely been seen in the clinic. Her blood pressures have been elevated and increasing on her last peri-natal visits. As the clinician, the decision is made to commence anti-hypertensive medical therapy. When assessing the patient for methods to reduce her blood pressure, the provider understands which of the following is contraindicated in this patient? A. Beta-blocker B. Triamterene-containing diuretic C. Central alpha-agonist D. ACE inhibitor

Correct Answer: D. ACE inhibitor ACE inhibitors are Pregnancy Class D drugs and are contraindicated in pregnancy. Incorrect Answers: A. In a 2013 systematic review of 13 population-based case-control or cohort studies examining the risk of congenital malformations associated with first-trimester oral beta-blocker exposure compared with no exposure, there was no overall increase in major congenital malformations. Labetalol has both alpha- and beta-adrenergic blocking activity and may preserve uteroplacental blood flow to a greater extent than traditional beta-blockers. B. Triamterene is Pregnancy Class C and is not contraindicated in pregnancy. C. Central alpha agonists are not contraindicated during pregnancy. Methyldopa is Pregnancy Class B/C.

A 73-year-old man is working in his garden when he develops sudden pain in his right calf and is unable to ambulate. He arrives at the clinic assisted by his daughter and complains of a hard, tight, painful calf. On exam, he has contractions of the gastrocnemius muscle. He notes this has been a recurrent problem that usually resolves with rest and massage. He has no edema, his capillary refill is normal, and pedal pulses are palpable. You order a chemistry panel to check his electrolytes and discuss preventative measures such as adequate hydration and various stretching exercises. He requests a "muscle relaxant" that can be used on an as-needed basis to mitigate the pain if the spasm recurs. Which of the following medications should never be prescribed to an elderly patient? A. Baclofen B. Tizanidine C. Metaxalone D. Carisoprodol

Correct Answer: D. Carisoprodol Carisoprodol is a centrally acting skeletal muscle relaxant used for the acute treatment of musculoskeletal pain. It should be avoided in elderly patients due to anticholinergic side effects that can cause delirium, sedation, and falls. This medication is on the Beers list because the risks outweigh any potential benefits of treatment. Due to its questionable efficacy at tolerable doses and high risk of serious side effects, carisoprodol should never be prescribed to elderly patients as a muscle relaxant. Incorrect Answers: A. Baclofen is a muscle relaxant and antispasmodic agent. Baclofen acts on the central nervous system (CNS) and commonly causes drowsiness, dizziness, and sedation as side effects. B. Tizanidine is a short-acting muscle relaxer that is used to treat spasticity by temporarily relaxing muscle tone. Common side effects include dry mouth, dizziness, and constipation. C. Metaxalone is a muscle relaxant that has no direct action on the contractile mechanism of striated muscle. The mechanism of action has not been established but may be due to CNS depression in general. The most common adverse reactions are drowsiness, dizziness, headaches, and gastrointestinal problems.

A 16-year old boy in New Hampshire presents with a painful, erythematous elbow that has gotten worse over the past week. He denies any constitutional symptoms such as fever, swollen lymph nodes, or weight loss. On physical exam, his right elbow is warm and tender to palpation. He denies any history of rashes. He often enjoys hiking in the woods. An arthrocentesis is performed which ruled out gout. Which of the following treatments should be initiated for this suspected disease? A. Azithromycin B. Ceftriaxone C. Corticosteroids D. Doxycycline

Correct Answer: D. Doxycycline Lyme disease is the most common tick-borne illness in the United States. It is caused by the bacterium Borrelia burgdorferi and is transmitted primarily by the deer tick (Ixodes scapularis or Ixodes pacificus on the West Coast). With untreated disease, the most common sites of extracutaneous involvement are the joints, nervous system, and cardiovascular system. Musculoskeletal symptoms are the most common extracutaneous manifestations of disseminated disease and can occur with early or late disease. Common manifestations of the early disease include transient oligoarticular symptoms of arthralgia or myalgia that may include joint swelling. Arthritis is usually a manifestation of late disease and occurs in up to 60% of untreated patients. Patients typically present 6 months after infection with joint pain and swelling, and synovial fluid findings suggest an inflammatory process. Intermittent inflammatory arthritis often begins as a migratory polyarticular process involving bursae, tendons, and joints, which evolves over 1-2 days into a monoarticular process involving the knee, ankle, and wrist, in decreasing frequency. When asked about symptoms after they have resolved, patients with Lyme disease are less likely to remember those symptoms that occurred before the monoarthritis. Polyarticular episodes may also occur. In patients without the neurologic disease, Lyme arthritis can usually be treated successfully with oral antibiotics with an extended treatment time of 28 days with doxycycline at 100mg BID. Patients with mild residual joint swelling after a recommended course of oral antibiotic therapy can be re-treated with another 4-week course of oral antibiotics. Patients whose arthritis fails to improve or worsens can be re-treated with a 2- to 4-week course of intravenous ceftriaxone. IDSA guidelines suggest that clinicians consider waiting several months before starting the second round of antibiotics, as joint inflammation tends to resolve slowly even when the infection has been eliminated.

Typically, the first teeth to erupt are the: A. Canines B. First molars C. Second molars D. Incisors

Correct Answer: D. Incisors The incisors typically erupt between 8 and 12 months. Incorrect Answers: A. Canines or cuspids erupt at 16 to 22 months. B. The first molars erupt at 13 to 19 months.C. Second molars erupt at 25 to 33 months.

A 15-year-old female presents for evaluation for painful menstrual cycle. She has a history of migraines with aura. She had her first menstrual cycle at the age of 12 years. Over the last few years, each period has lasted 6-7 days and occurs once every 29 days. Over the last 6 months, she has experienced pain in her lower abdominal and upper thighs during the first few days of her menstrual cycle. She has missed 2-3 days of school each month due to the pain. She denies sexual activity. She has not tried any over the counter medications for the pain. Which of the following would be the first-line treatment for her condition? A. Norgestrel 0.3mg/ethinyl estradiol 30ug Q day for 30 days B. Transdermal patch containing 6 mg norelgestromin and 75mg EE C. Acetaminophen 500mg Q4-6 hours D. Naproxen 500mg once, then 250 mg Q6-8 hours

Correct Answer: D. Naproxen 500mg once, then 250 mg Q6-8 hoursExplanation: The adolescent in this question is suffering from primary dysmenorrhea. Dysmenorrhea is common in young women around the world and occurs in most adolescents. Primary dysmenorrhea will not usually present until two to three years after menarche. Menstrual cycles in the first few years after menarche are anovulatory. Pain with dysmenorrhea is often associated with headaches, nausea, and diarrhea. These symptoms are caused by the secretion of prostaglandins from the uterus after the anovulatory cycle, resulting in increased uterine contractions and upregulation of pain receptors. Naproxen and ibuprofen are non-steroidal anti-inflammatory drugs (NSAIDs) that inhibit cyclooxygenase. Cyclooxygenase is an enzyme necessary to produce prostaglandins. Incorrect Answers: A. In adolescents who have not had an improvement in pain after three menstrual cycles on NSAIDs oral contraceptives would be the next step. Since this patient has migraines with aura combined oral contraceptives are contraindicated. B. In adolescents who have not had an improvement in pain after three menstrual cycles on NSAIDs hormonal contraceptives would be the next step. Since this patient has a history of migraines with aura contraceptives that contain estrogen are contraindicated. C. Acetaminophen will not inhibit the production of prostaglandins making it an ineffective treatment for dysmenorrhea.

A 72-year-old woman with a history of hypertension comes to the emergency department with difficulty breathing. She has experienced exertional shortness of breath over the last few weeks and worsening symptoms at night when she lies flat. Echocardiography demonstrates diffuse hypertrophy of the left ventricle and an ejection fraction of 65%. In this patient population, which of the following confers a mortality benefit? A. Beta-blockers B. Diuretics C. Spironolactone D. ACE inhibitors E. No agent has shown a mortality benefit in this population

Correct Answer: E. No agent has shown a mortality benefit in this population Heart failure with a preserved ejection fraction is characterized by impaired diastolic relaxation, leading to increased left ventricular end-diastolic pressure and symptoms of left-sided heart failure. Decreasing afterload by treating hypertension and treating fluid overload with diuresis serve as the primary goals of treatment in this patient population. Medications with a significant mortality benefit in patients with reduced ejection fraction such as beta-blockers, ACE inhibitors, angiotensin receptor blockers, and spironolactone do not have the mortality benefits seen in heart failure with a preserved ejection fraction.

A 56-year-old man with a history of aortic valve repair for aortic stenosis presents to his primary care provider for a routine visit. The patient feels well and exercises daily without symptoms. The physician recommends a routine screening colonoscopy with a potential biopsy of any abnormal lesions. Which of the following endocarditis prophylaxis regimens is appropriate for this patient? A. Oral amoxicillin B. Intravenous ceftriaxone C. Oral Augmentin D. Oral clindamycin E. No prophylaxis indicated

Correct Answer: E. No prophylaxis indicated Prophylaxis for infective endocarditis is generally recommended for patients at a high risk of developing endocarditis, typically with structurally abnormal hearts undergoing procedures with a high risk for transient bacteremia. For patients meeting these criteria, oral amoxicillin is typically used unless there is a concern for resistant organisms, in which case a cephalosporin or vancomycin may be appropriate. Patients requiring antibiotic prophylaxis include those with prosthetic heart valves, cardiac valve repairs using prosthetic material, a history of infective endocarditis, an unrepaired cyanotic congenital heart lesion, a repaired congenital lesion with residual shunts, or valvular regurgitation in the setting of a transplanted heart. In this population, antibiotic prophylaxis is only required during invasive dental procedures, which includes routine cleaning, respiratory tract procedures involving manipulation of the mucosa, skin or soft tissue procedures involving infected tissue, or cardiac surgery with prosthetic material. Antibiotics are typically not indicated for gastrointestinal or genitourinary procedures. Incorrect Answers: A. Oral amoxicillin is typically the antibiotic of choice for endocarditis prophylaxis. For a low-risk procedure, prophylaxis is not required. B. Cephalosporins may be appropriate for prophylaxis in high-risk patients undergoing respiratory procedures. C. This agent is not typically used as endocarditis prophylaxis.

You have been following a patient with type 2 diabetes for two years. Over the last several days she has been injecting exenatide 5 mcg twice daily about 45 minutes before meals. Her current medications include metformin 1000mg twice daily, exenatide 5 mcg twice daily, glyburide 5mg twice daily, Lisinopril 20mg daily, simvastatin 10mg daily and aspirin 81mg daily. She calls your office and reports symptoms of hypoglycemia that begins a couple of hours after eating. What is your next step in patient care? A. Decrease exenatide dose B. Decrease glyburide dose C. Encourage her to consume more at meals D. Decrease metformin dose

Correct Answer: B. Decrease glyburide dose By decreasing the glyburide dose, the potential for hypoglycemia is decreased (see explanation for choice A). Incorrect Answers: A. Exenatide alone has an 11% association with hypoglycemia. However, when combined with a sulfonylurea such as Glyburide, the association increases anywhere from 14-36%. C. This will not help her hypoglycemic episodes. D. Metformin is associated with the risk of hypoglycemia. However, this risk is not impacted by the use of exenatide.

A 19-year-old patient with tricuspid atresia status post right atrial to pulmonary artery Fontan connection requests reversible contraception. She has had a thrombus in her right atrium 2 years ago, and she is taking warfarin anticoagulation. She is unmarried and has had 2 partners. Which of the following should you recommend? A. Hysterectomy B. Depo-Provera intramuscular injection C. Natural family planning D. Low-dose estrogen cyclical oral contraception

Correct Answer: B. Depo-Provera intramuscular injection Depo-Provera injection provides the best option for this patient, though there is some risk of hematoma at the injection site. Incorrect Answers: (A) A hysterectomy is not reversible.(C) Natural family planning is not as effective as the other methods mentioned with typical use. (D)Estrogen containing oral contraception would not be a good choice in a patient at risk for thrombus. Vital Concepts: Depo-Provera injection provides the best option for this patient, though there is some risk of hematoma at the injection site. References: U.S. Medical Eligibility Criteria for Contraceptive Use, 2016.

A 68-year-old male presents with an acute onset of a red, swollen, painful knee. He has diabetes, hyperlipidemia, and chronic kidney disease. He denies fevers, chills, and trauma. His vitals are as follows: Temp = 98°F HR = 70BP = 140/80 RR = 14 On exam, he has a joint effusion with erythema and generalized tenderness. There is no streaking. Fluid from the knee aspiration reveals a cell count of <1000, and his culture 72 hours later shows no growth. What is the most likely diagnosis? A. Prepatellar bursitis B. Gout C. Septic arthritis D. Meniscal tear

Correct Answer: B. Gout The most likely diagnosis is gout. The sudden onset and co-morbidities are highly suspicious for gout. The white blood cell count is not high enough to suggest a bacterial joint infection, but it does suggest an inflammatory process.Of the answers given, gout is most likely. The joint aspirate in a patient with acute gout also contains negatively birefringent needle-shaped crystals of monosodium urate. Incorrect Answers: A. Prepatellar bursitis can be difficult to distinguish from other diagnoses, as it causes redness and swelling in the knee area. As the bursa is extra-articular, it does not cause a knee effusion. C. Gout and septic arthritis can present very similarly clinically. The best management is to aspirate the knee to obtain a cell count. While joint-fluid aspiration only grows a specific organism <50% of the time, his cell count makes septic joint unlikely. Bacterial joint infections usually produce purulent effusion with leukocyte counts (most of which are neutrophils) of 50,000 to 150,000 cells/mm3. D. Meniscal tears can be insidious in this age population and can cause joint effusions. However, they are unlikely to cause erythema.

The nurse practitioner observes lid lag in a client with: A. Myasthenia gravis B. Hyperthyroidism C. Hordeolum D. Chalazion

Correct Answer: B. Hyperthyroidism Lid lag is seen in hyperthyroidism. It is due to sympathetic overactivity. If the lid margin falls above the limbus so that some sclera is visible, hyperthyroidism may be present. Incorrect Answers: A. Ptosis is a drooping lid margin that falls at the pupil or below and may indicate myasthenia gravis. C. A localized infection of the small glands around the eyelashes in the hair follicle at the lid margin is called Hordeolum. D. A Chalazion is an inflammation or cyst of the meibomian glands, which lie within the posterior portion of the eyelid. Vital Concepts: Lid lag is seen in hyperthyroidism. It is due to sympathetic overactivity. If the lid margin falls above the limbus so that some sclera is visible, hyperthyroidism may be present.

Which of the following is true regarding febrile seizures in children? A. Two thirds of children will have a second seizure B. It is normal for a child to lose conscious during a febrile seizure C. Febrile seizures will lead to epilepsy later in the child's life D. Febrile seizures are not dangerous once they pass

Correct Answer: B. It is normal for a child to lose conscious during a febrile seizure This is normal. Febrile seizures are usually tonic-clonic seizures and loss of consciousness accompanied by bilateral limb shaking is common. Incorrect Answers: A. 30% of children will have a second seizure, 50% will have another seizure if they have a second. C. According to the AAP, febrile seizures do not ensure that a child will develop epilepsy, it only slightly increases the risk. D. Children should be examined by a health care professional to determine the cause of the fever. It is important to rule out meningitis.

A 16-year-old male presents at the office with his parents for a wellness check. What are the most common locations of osteochondritis dessicans (OCD) lesions in pediatric patients? A. Cervical, thoracic, and lumbar spine B. Knee, ankle, and elbow C. Pelvis, toe, and nasal plate D. Eye, ear, and palate

Correct Answer: B. Knee, ankle, and elbow Knee, ankle, and elbow are the most common locations of OCD lesions. Many of the other choices are not potential locations for OCDs, particularly the cartilaginous areas like the ear. OCD is usually due to repetitive trauma or overuse in athletes. It occurs more often in boys than in girls. The knee is the site affected most commonly, followed by the elbow and ankle. Usually, the localized segment of subchondral bone separated from the articular cartilage can be seen on X-ray. Incorrect Answers: (A), (C), (D) are not the correct locations for OCD lesions.

An adult client presents to the nurse practitioner's office with a white plaque near the base of the tongue. The NP notes that the plaque does not wipe off and makes which of the following assessments? A. Hemangioma B. Leukoplakia C. Papilloma D. Erythroplasia

Correct Answer: B. Leukoplakia Leukoplakia is a white patch present on the oral mucosa that cannot be rubbed off. Incorrect Answers: A. Hemangiomas are benign blood vessel proliferation of the lips, tongue, or buccal mucosa.C. Papillomae are benign verrucous lesions that are manifestations of human papillomavirus infection. D. Erythroplasia is an asymptomatic, red, velvety lesion of the mouth. Vital Concepts: Leukoplakia is a white patch present on the oral mucosa that cannot be rubbed off.

A 33-year-old woman complains of mild pain and swelling in her wrist. The pain is located along the radial side of the wrist. She reports reduced sensation along the radial side of her hand with accompanying numbness and tingling. In addition, she notes a nodule on the dorsal aspect of her wrist. She does not know exactly when the pain and discomfort started and cannot recall any traumatic or inciting event. She has taken pain medication with no improvement in her symptoms. The patient's temperature is 98.3 degrees Fahrenheit, heart rate is 80 beats/minute, respiratory rate is 16 breaths/minute, and blood pressure is 130/90 mmHg. On physical exam, the nodule is firm, tender, and transilluminates. Which of the following diagnostic tests is indicated for this patient? A. X-ray B. MRI C. Arthroscopic exam D. No imaging needed

Correct Answer: B. MRI A ganglion cyst is a cystic swelling typically found on the dorsal side of the wrist. These cysts are thought to arise from herniation of synovial tissue from a joint capsule or tendon sheath. They present with chronic wrist pain and may be accompanied by the coolness of the hand, numbness/tingling, and possible paresthesias. Classically on exam, the mass itself is mobile, tender to palpation, and transilluminates. Most cases of ganglion cysts are diagnosed by history and physical exam, so no further imaging is needed. Transillumination will differentiate a cystic lesion from a solid mass, but an ultrasound is more reliable. This patient presents with pain and neurovascular symptoms, indicating possible structural damage to the cyst invading deeper tissues. In these cases, MRI will allow the best visualization of the tissue and assess neurovascular damage more accurately and rule out other potential causes. Incorrect Answers: (A) An X-ray will not provide the needed visualization of the tissue.(C) A diagnostic test is needed for this patient as the patient presents with pain and neurovascular symptoms. (D) Since the patient presents with pain and neurovascular symptoms, further imaging is needed.

An otherwise-healthy 32-year-old G2P2 calls her primary obstetrician in January, 8 days after an uncomplicated vaginal delivery at 39 weeks. She has had fevers to 101.9°F, cough, and muscle aches. She thinks that one of her friends may have had a similar illness the past week. She received the influenza vaccine during her pregnancy. She has no vaginal bleeding or discharge and no abdominal pain besides the aforementioned myalgias. She is breastfeeding. She has had a slight loss of appetite but otherwise no gastrointestinal upset. Which of the following steps is indicated? A. Conservative management of symptoms without antiviral medication B. Maternal-neonatal separation C. Temporary cessation of breastfeeding (i.e. dumping breast milk and formula feeding) D. Admission to hospital

Correct Answer: B. Maternal-neonatal separation Women should be treated empirically with antivirals for influenza-like illness up to 2 weeks postpartum. These women should be separated from their neonates, but breastmilk can be expressed and fed to the newborn by bottle by a healthy adult during the separation period. Pregnant women with influenza infection are at significantly higher risk of serious morbidity and death than the general population. The Centers for Disease Control and Prevention (CDC) recommend that all pregnant women receive the inactivated influenza vaccine, regardless of trimester. The CDC additionally recommends separation of neonates from mothers with suspected influenza infection; one protocol is to continue separation until the mother is 24 hours afebrile and has received antivirals for 48 hours. Incorrect Answers: A. A case series suggests that the period of increased risk of morbidity and mortality extends to the early postpartum period. As such, the CDC also recommends that women be empirically treated for influenza up to 2 weeks postpartum. C. There is no evidence of harm to neonates by breastfeeding mothers taking oseltamivir; breastmilk can be expressed and fed to the newborn by bottle by a healthy adult during the separation period. D. Pregnant or postpartum patients should be evaluated in the hospital if they have significant respiratory complaints, underlying medical disease, or are unable to stay hydrated. Otherwise, uncomplicated cases may be triaged without a hospital visit depending on the provider's comfort, knowledge of the patient, and likelihood of exposing other patients to influenza.

A 20-year-old male with no PMH presents with shoulder pain that has occurred for more than 8 months. He has been a baseball pitcher for several years and started feeling tenderness around his shoulder region 8 months ago. He denies any trauma but states that several months ago, he felt a pull on his shoulder when he threw a screwball, resulting in pain after the incident. The pain has not progressed since the incident and improved with rest and NSAIDs. The pain is not related to temperature or time of day and did improve with lidocaine injection. The patient's temperature = 98°F, HR = 70, RR = 14, and BP = 110/75. Physical exam shows tremendous difficulty in lifting his left arm above his head accompanied by pain. There is also a clicking sound when the arm is raised. Which of the following is NOT consistent with this patient's condition? A. Patient's sport B. More common in younger patients C. Difficulty in lifting his arm D. Presence of clicking sound

Correct Answer: B. More common in younger patients Older people tend to get rotator cuff tendinitis more than younger people. Rotator cuff tendinitis is the swelling of the rotator cuff tendons. It typically is caused by repetitive activities associated with overuse of the arm and shoulder, especially in physical activities such as volleyball or baseball. Symptoms involve shoulder weakness and difficulty lifting the arm over the head for several months to a few years. In addition, there may be pain and swelling in the front of the shoulder, a clicking sound when the arm is raised, and stiffness. Resting and icing the shoulder along with ibuprofen treatment should improve the symptoms. Our patient has long chronicity of symptoms, clicking sound, difficulty lifting the shoulder, and repetitive motions with baseball. Incorrect Answers: (A), (C), (D) are all consistent with rotator cuff tendinitis.

You are examining an elderly woman. There is a glossy white circle around the cornea of one of her eyes, and the pupil has a decreased reaction to the light reflex. There is a history of presbyopia. Correct interpretation of these findings is: A. Beginning development of cataracts with a significant decrease in visual acuity B. Normal change in the eye as a result of the aging process C. A decrease in depth perception and the early eye changes associated with glaucoma D. Visual changes caused by long-term treatment with digoxin

Correct Answer: B. Normal change in the eye as a result of the aging process With cataracts, there is a clouding of the lens, not a circle around the cornea. The circle around the cornea is arcus senilis, which is normal in the geriatric patient. Incorrect Answers: A. The visual acuity of the client cannot be determined from the information provided. C. There are no observable pupillary changes in the beginning stages of glaucoma. D. Digoxin toxicity can lead to visual disturbances (blurring, central scotomas, glare effects, altered color perception), but the description in the question is more consistent with arcus senilis. Vital Concepts: With cataracts, there is a clouding of the lens, not a circle around the cornea. The circle around the cornea is arcus senilis, which is normal in the geriatric patient.

A 14-year-old female basketball player presents with anterior knee pain that is exacerbated by jumping. She denies any specific injury. There is no effusion, joint-line tenderness, or instability. She has tenderness to palpation over the tibial tubercle. What is the most likely diagnosis, and what is the appropriate management? A. Meniscal tear and arthroscopic surgery B. Osgood-Schlatter disease and rest, ice, and anti-inflammatories C. Anterior cruciate ligament tear and physical therapy D. Patellofemoral pain syndrome and realignment surgery

Correct Answer: B. Osgood-Schlatter disease and rest, ice, and anti-inflammatories Osgood-Schlatter is the eponym used for anterior knee pain that is often found in adolescents. As they grow, their bones lengthen faster than their tendons, which can lead to apophysitis of the tibial tubercle. Management is rest from activity, stretching, and anti-inflammatories. Pain often resolves once growth has ceased. Occasionally, surgery is required if the apophysis becomes avulsed. Incorrect Answers: A. Effusions are common in meniscal injuries. There is no history of twisting or specific injury. She has no joint-line tenderness. In addition, not all meniscal injuries require surgery. C. Effusions are common with ACL tears. The typical treatment is reconstructive surgery if there are persistent symptoms of instability. D. PFPS symptoms include tenderness on the patellar facets and a positive grind test. While some of the symptoms are similar in PFPS and Osgood-Schlatter disease, the appropriate treatment for PFPS is physical therapy, not realignment surgery.

A 51-year-old presents to the office with a known history of seizure disorder. Patient reports taking valproic acid. Which of the following is a potential adverse effect of valproic acid? A. Gingival hyperplasia B. Pancreatitis C. Kidney stones D. Weight loss

Correct Answer: B. Pancreatitis Pancreatitis is one of the most serious adverse effects that can be caused by valproic acid. There is a boxed warning concerning pancreatitis, in addition to boxed warnings concerning hepatotoxicity and fetal risk. Incorrect Answers: A. Gingival hyperplasia is a side effect of phenytoin.C. and D. Kidney stones and weight loss are side effects of zonisamide or topiramate.

A 62-year-old man with a past medical history of hypertension, chronic obstructive pulmonary disease (COPD), and diabetes mellitus type II (DM II) presents to the primary care clinic for a 2-week follow-up after being discharged from the hospital for a COPD exacerbation. He has no complaints and is compliant with his home medications. His vitals are temperature 98.8 degrees Fahrenheit, heart rate 88 beats per minute, respiratory rate 18 breaths per minute, and blood pressure 138/92 mmHg. Physical exam is unremarkable. The patient received vaccines through his 20s but not since then. Which of the following vaccinations is indicated at this visit? A. Pneumococcal conjugate (PCV13) vaccine B. Pneumococcal polysaccharide (PPSV23) vaccine C. Meningococcal vaccine D. Hepatitis A vaccine

Correct Answer: B. Pneumococcal polysaccharide (PPSV23) vaccine There are two types of pneumococcal vaccine, a 13-valent conjugate vaccine (PCV13) and a 23-valent polysaccharide vaccine (PPSV23), with 13 and 23 corresponding to the number of pneumococcal strains covered. PPSV23 is indicated for all adults ages 65 years or older. It is also indicated for adults <65 years old (like this patient) with chronic lung conditions (e.g. COPD, emphysema, asthma), chronic cardiovascular diseases, DM II, chronic renal disease, chronic liver diseases, and chronic alcohol use disorder, as well as immunocompromising conditions (e.g. HIV, nephrotic syndrome). This patient is <65 years old and has COPD, DM II, and hypertension. Both COPD and DM II qualify him for early (<65 years old) PPSV23. When indicated, it is also recommended to administer PPSV23 to patients without a reliable immunization record. Incorrect Answers: A. For anyone ages 19-65 with asplenia, an immunocompromising condition (e.g. chronic renal failure, nephrotic syndrome), cerebrospinal fluid leaks, or cochlear implants, PCV13 is recommended. When indicated, also administer PCV13 to patients without a reliable vaccination record. For all patients over 65 years old without the aforementioned conditions, PCV13 administration should be decided based on shared decision making between the patient and provider.

A medical provider is evaluating a 78-year-old female with depression. She suspects the patient may have Parkinson's disease. Which of the following is consistent with that diagnosis? A. Broad-based gait B. Postural instability C. Increased facial movement D. Writhing movement

Correct Answer: B. Postural instability Parkinson's disease is a movement disorder caused by destruction of dopaminergic neurons in the substantia nigra. Patients frequently exhibit a pill-rolling tremor, shuffling gait, difficulty initiating movement, postural instability, and a mask-like facies (decreased facial movement.) There are also non-motor manifestations of Parkinson's disease, which include autonomic dysfunction, gastrointestinal dysfunction, rhinorrhea, sleep disturbances, fatigue, mood disorders, cognitive dysfunction, dementia, psychosis, and hallucinations. Incorrect Answers: A. Broad-based gait. Broad-based gait is seen in cerebellar disease and acute alcohol intoxication.C. Increased facial movement. Involuntary increased facial movements occur with tardive dyskinesias. D. Writhing movement. Writhing movements occur with Huntington's chorea.

An examiner is preparing to examine the eyes of an adult client. In order to examine the optic disc and retinal vessels, the practitioner uses which aperture on the ophthalmoscope? A. Small aperture B. Red-free filter C. Slit D. Grid

Correct Answer: B. Red-free filter The red-free filter is used to examine the optic disc for pallor and vascular changes, as well as to detect retinal hemorrhages. Incorrect Answers: A. The small aperture is used to detect retinal hemorrhages.C. The slit is used for the anterior eye.D. The grid is used for estimating the size of the lesions found in the frontal area. Vital Concepts: The red-free filter is used to examine the optic disc for pallor and vascular changes, as well as to detect retinal hemorrhages.

A 35-year-old male presents for complaints of 1 week of left testicular discomfort and swelling. He says he was carrying a pail of paint up a ladder when he suddenly felt pain in the region. That evening, he noticed a swelling there that was only mildly uncomfortable. He denies vomiting, fever, urinary symptoms, direct trauma, or other symptoms. He has no medical history except a tonsillectomy as a child. On arrival, his vitals are BP 124/84, HR 65, RR 16, Temp 97.9, and Sat 100% on RA. On exam, there is moderate swelling of his left scrotum, which is non-tender but more prominent when he is standing and nearly resolves while lying flat. Cremaster reflex is present. The rest of his exam is unremarkable. What should be done for him? A. Order STAT testicular sonogram B. Refer to surgeon over the next week or so C. Check UA and treat with Cipro for 2 weeks D. Refer him to ER for immediate surgical treatment

Correct Answer: B. Refer to surgeon over the next week or so This is a reducible indirect inguinal hernia. It is indirect since it protrudes into his scrotum, but the important thing is that it reduces. In this case, resolution occurs nearly spontaneously and needs only elective or semi- urgent repair if it is really bothering the patient. Incorrect Answers: A. This is not a swollen testes, and the cremaster reflex is present, so there is no need for urgent testicuar sono or manual detorsion. C. This is also not epididymitis, so he does not need a UA or antibiotics. D. If it were incarcerated (unreducible) or strangulated (shows signs of ischaemia to the bowel, rock hard, extremely tender, febrile, and vomiting), then he would need emergent repair, and sending him to the ER would be appropriate.

All of the following are responsible for bacterial AOM in children EXCEPT? A. Streptococcus pneumoniae B. Staphylococcus aureus C. Moraxella catarrhalis D. Haemophilus influenzae

Correct Answer: B. Staphylococcus aureus The three most common pathogens in AOM: Streptococcus pneumoniae, Moraxella catarrhalis, and Haemophilus influenzae. Amoxicillin or Amoxicillin-clavulanate arethe first-line treatment for AOM. S. aureus is rarely a causative agent in AOM. Incorrect Answers: (A), (C), (D) are correct organisms responsible for AOM.

The most common pathogen seen in an otitis media infections is: A. Moraxella catarrhalis B. Streptococcus pneumoniae C. Streptococcus pyogenes D. Pseudomonas aeroginosa

Correct Answer: B. Streptococcus pneumoniae The most common diseases associated with Haemophilus influenza are pneumonia, bacteremia, and meningitis. The most common bacterial pathogen in AOM is Streptococcus pneumoniae, followed by nontypeable Haemophilus influenzae and Moraxella (Branhamella) catarrhalis. These three organisms are responsible for more than 95% of all AOM cases with a bacterial etiology. Incorrect Answers: A. Moxarella catarrhalis is associated with otitis media, but it is not the most common pathogen. C. Streptococcus pyogenes is associated with pharyngitis, scarlet fever, impetigo, or cellulitis.D. Pseudomonas aeroginosa is the most common bacterium in otitis externa infections.

A 29 year-old woman presents with pain in the right ear beginning one week ago. This was followed by the gradual onset of weakness throughout the right side of her face. She also describes an oversensitivity to noise in the right ear and impaired taste. Which of the following is true? A. The lesion of the facial nerve must be distal to the stylomastoid foramen B. The lesion of the facial nerve must affect the chorda tympani C. Facial reflexes will be spared D. Emotional facial responses will be spared, while only voluntary motions will be affected E. Weakness is unlikely to fully resolve

Correct Answer: B. The lesion of the facial nerve must affect the chorda tympani The patient likely presents with Bell's palsy, an acute, unilateral, infranuclear facial nerve palsy. The onset typically consists of pain in or behind the ispilateral ear, followed by worsening facial weakness over several days. Patients may also report impaired taste or hyperacusis, depending upon the regions of the nerve that are involved. MRI may reveal contrast enhancement of the facial nerve. Bell's palsy is often viral in etiology (i.e. herpes simplex). In many cases, the cause remains unknown. Other less common etiologies include sarcoidosis and Lyme disease. The lower motor neuron impairment of Bell's palsy must be distinguished from upper motor neuron dysfunction. The dorsal aspect of the facial motor nucleus receives input from bilateral cortex, while neurons in the ventral aspect of the nucleus receive primarily contralateral cortical input. The result is that the muscles of the upper face are under bilateral cortical control, while the lower facial musculature is under contralateral cortical control. The clinical consequence is that unilateral cortical lesions may spare facial emotional responses and reflexes due to their bilateral cortical control. The more distal lower motor lesions would impair facial nerve function more broadly, voluntary movements, emotional responses, and reflexes equally. A branch of the facial nerve, the chorda typmpani, carries efferents to the sublingual and submandibular glands, to control salivation. It also contains taste fibers from the anterior two-thirds of the tongue. Decreased taste sensation in the setting of Bell's palsy suggests involvement of the chorda tympani. Incorrect Answers: A. The facial nerve exits the skull at the stylomastoid foramen, where the nerve branches to innervate the facial muscles. Of note, the only facial muscle not innervated by these branches is the levator palpebra superioris, controlled by the oculomotor nerve. An isolated lesion at this level would not explain hyperacusis

The NP is caring for an Rh-negative pregnant patient. The NP knows that this indicates: A. The fetus has autoantibodies against maternal RBCs. B. The mother will develop antibodies if exposed to fetal Rh-positive red blood cells. C. The fetus will have autoantibodies that cause damage to fetal blood cells. D. The mother's Rh factor will attack fetal blood cells.

Correct Answer: B. The mother will develop antibodies if exposed to fetal Rh-positive red blood cells. The mother's immune system will develop antibodies when exposed to fetal blood cells that are Rh-positive. The infant of an Rh-negative mother can be Rh-positive or negative, depending upon the father's blood type. Exposures can occur during trauma, surgery, or delivery. The maternal antibodies will destroy the Rh-positive red blood cells of the fetus, causing hemolytic anemia. The NP should administer Rho(D) immune globulin to the Rh-negative mother at 28 weeks in the pregnancy and after the birth if the newborn is Rh-positive. Incorrect Answers: A. Autoantibodies, which are antibodies produced by the immune system to attack the individual's own tissue, would, in this case, refer to antibodies against the tissue of the fetus. This is incorrect. C. Maternal antibodies are produced and act against the Rh-positive red cells of the fetus. D. Rh factor is a protein on the surface of the red blood cell. Antibodies are formed in reaction to the recognition of Rh factor on the fetal red blood cells. Rh factor does not attack blood cells.

A nurse practitioner is sued by a patient she saw in her clinic. When considering a malpractice lawsuit, which of the following is correct? A. The nurse practitioner is the plaintiff in this case B. The nurse practitioner is the defendant in the case C. During the trial phase, medical records will be requested D. If damages are awarded, they will only include reasonable fees for medical expenses

Correct Answer: B. The nurse practitioner is the defendant in the case Medical malpractice lawsuits are filed by patients or on behalf of the patient or other party claiming damage. This person is the plaintiff and the healthcare provider is the defendant in the case. A plaintiff must prove all of the following four elements: • A duty was owed to the plaintiff by the defendant (legal duty) • The duty was breached (standard of care was not met, etc.)• This breach of duty caused an injury (proximate cause)• Damage occurred. A lawsuit is filed in an appropriate court and the discovery phase begins with requests for medical records, expert opinions, and depositions. The plaintiff has the burden of proof. The case will either enter a trial phase or settle out of court. Lawsuits can be dismissed or damages can be awarded for physical, emotional, and mental harm. Incorrect Answers: A. The suing party is the plaintiff — that's the patientC. Medical records are requested in the discovery periodD. Damages can be awarded for physical, emotional, and mental harm in addition to medical fees

A 33-year-old woman is brought in by her husband for "erratic behavior." She reports that a month ago, she was depressed, but one day she figured out that what she needed was a change in her medication dose. She halved the medication, and she felt so good that she halved it again the following day. "I feel superfidiciously great!" she crows. Her husband relates that she went shopping again this morning and picked up 2 bottles of expensive perfume, 1 bottle of $400 cologne, and an armload of designer clothes. Which of the following is a medication she was likely on? A. Fluoxetine (Prozac) B. Valproate sodium (Depacon) C. Bupropion (Wellbutrin) D. Mirtazapine (Remeron)

Correct Answer: B. Valproate sodium (Depacon) Both lithium and valproate sodium are treatments for bipolar disease, and a reduction in her dosage is likely what brought her into a manic state. None of the other medications listed is the first-line treatment for bipolar disease. If she had doubled a dose of medication instead of reducing it, fluoxetine would be the likely culprit to induce mania. Incorrect Answers: (A), (C), (D) None of the other medications listed is the first-line treatment for bipolar disease.

What is the confirmatory test most commonly used when an initial screening test for HIV is positive? A. Viral load (PCR) B. Western Blot C. ELISA D. CD 4 count

Correct Answer: B. Western Blot Western blot is used to confirm HIV infection and is generally used after a positive ELISA testing. A combination antibody/antigen test may also be used because they can detect HIV p24 antigen before antibodies are present. Incorrect Answers: A. Viral load reflects the plasma viral load (number of copies per milliliter of plasma). Lower number reflects slower progression of disease. Can be used to diagnose acute HIV infection. C. ELISA is used as an initial screening test. While sensitivity of ELISA is high, Western blot is used to confirm positive tests. It may take weeks to 3 months from initial infection for an infected person to seroconvert and for ELISA to be positive. This is why the antigen test is helpful. D. CD4 counts measures the body's immune system and degree of immunosuppression. Higher count reflects higher degree of immune function and slower disease progress. CD4 counts and viral loads are routinely monitored to determine treatment effectiveness.

A client with RA (Rheumatoid Arthritis) presents for follow-up. What is the best evaluative question the nurse practitioner can ask that will help determine the severity of disease? A. "Were you able to drive the car to your appointment today?" B. "Were you able to fix dinner last night?" C. "How long does it take for your joints to loosen up after you get up in the morning?" D. "How many pounds can you carry?"

Correct Answer: C. "How long does it take for your joints to loosen up after you get up in the morning?" Morning stiffness on activity and the length of time it takes for maximal improvement are two of the American Rheumatism Association Classification criteria for RA (Rheumatoid Arthritis) and can be used to measure the effects of treatment. Incorrect Answers: A. B. D. The other questions are good indicators of quality of activity of daily living but do not give a full, overall, measurable picture of the client's discomfort. Vital Concepts: Morning stiffness on activity and the length of time it takes for maximal improvement are two of the American Rheumatism Association Classification criteria for RA (Rheumatoid Arthritis) and can be used to measure the effects of treatment.

A 32-year-old male presents with about 1 month of cough, intermittent chills, subjective fevers, myalgias, and anorexia. He says he thought he had just caught a cold, but he "just can't shake it." He reports no other past medical history and only takes OTC pain relievers occasionally for the symptoms. He smokes a few cigarettes a week in social settings. He denies GI and urinary symptoms. He denies recent travel or camping and has had no known sick contacts. Vitals are BP 130/80, HR 98, RR 18, Sat 96% on RA, and Temp 99.9 F. Exam reveals mild skin pallor, slightly injected conjunctiva, cobblestoning of the pharynx, and a few coarse rhonchi in the lungs. His abdomen is soft and nontender. Rapid CBC shows WBC 2.8, Hgb 15, and Platelets 120. The chest X-ray appears normal. What should be done next for this patient? A. Draw full metabolic panel of labs and refer to Infectious Disease B. Draw blood cultures, lyme screen, and monospot C. Ask about HIV risk factors D. Start 1-week course of levofloxicin

Correct Answer: C. Ask about HIV risk factors This is a typical presentation of a prolonged viral illness. Although it can be caused by mononucleosis, tick- borne illness, and other things, it is common for the acute phase of HIV infection to present this way. The syndrome is caused by the surge in the viral load levels and temporary suppression of T cells. Eventually, the immune response mounts and the viral levels drop, and symptoms resolve. CD4 levels will usually drop over a period of years until they reach a critical level of 200mcL, where opportunistic infections tend to occur. It is critical to get more history in these patients at the time of onset to direct the workup. Incorrect Answers: (A) The patients should be assessed thoroughly prior to diagnostic testing.(B) The patients should be assessed thoroughly prior to diagnostic testing.(C) The patients should be assessed thoroughly prior to initiating any treatment.

A 28-year-old female athletic swimmer with no significant past medical problems presents to your office with a 4-day history of right ear itching and pain. She also has noticed some cloudy discharge coming out of it. Her vitals were within normal limits at this visit. Physical exam was significant for tenderness when the auricle was manipulated. The ear canal was narrowed and erythematous, and gray-colored discharge was observed. The tympanic membrane was partially visualized and it was mobile with pneumatic insufflation. What is the most appropriate treatment for this patient? A. Oral ciprofloxacin B. Hydrocortisone cream C. Cipro HC (Ciprofloxacin and Hydrocortisone) D. Saline irrigation alone

Correct Answer: C. Cipro HC (Ciprofloxacin and Hydrocortisone) This vignette is the classic presentation for otitis externa (OE), or swimmer's ear, which is an infection of the external auditory canal. If the infection extends to the skull base, it is known as malignant otitis externa. However, it is usually limited to the external auditory canal. In mild OE, patients suffer from minimal discomfort, mild pruritis, and minimal edema in the external auditory canal. In moderate disease, in addition to intermediate pain and pruritis, the ear canal is partially occluded (like the patient in the above scenario). In severe OE, the canal is usually completely occluded due to swelling, along with intense pain and pruritis. Classifying OE helps guide management. In general, management includes maintaining ear canal hygiene, treatment of the infection, and analgesia. Cleaning the ear canal allows for the treatment (ear drops) to be effective. Mild infections are usually addressed with an acetic acid eardrop solution and a steroid cream. In moderate infections, acidic solutions containing a steroid and an appropriate antibiotic are required. Since Staphylococcus aureus and Pseudomonas aeruginosa are the main offenders, a topical eardrop solution containing ciprofloxacin and a steroid is the preferred 1st line treatment. Oral antibiotics are usually reserved for patients with severe infections. Incorrect Answers: A. While ciprofloxacin covers both Staphylococcus aureus and Pseudomonas aeruginosa, oral antibiotics are not needed for moderate OE as explained above. B. A steroid cream alone is not sufficient for moderate OE. Even in mild OE, a steroid should be accompanied by an acidifying solution, such as acetic acid. D. Saline irrigation is an important 1st step in the treatment of OE in order to remove debris and allow the ear drops to be effective. However, it is not alone sufficient to address OE at any stage.

A 49-year-old female presents to the office with a history of breast cancer who is having frequent hot flashes who already tried first-line lifestyle changes without improvement? What is the best treatment for this patient? A. Hormonal therapy with estrogen and progesterone B. Bioidentical hormones C. Citalopram D. Black cohosh

Correct Answer: C. Citalopram Selective serotonin reuptake inhibitors (SSRI)s are the most effective pharmacologic choice for the vasomotor symptoms of menopause. Citalopram is an SSRI that has good evidence of efficacy against these symptoms and is typically well-tolerated. Incorrect Answers: A. Women with a history of breast cancer are not candidates for hormone replacement therapy. B. "Bioidentical hormones" are compounded mixtures of multiple hormones. There is no evidence to support their use at this time, and again, hormone treatment would be contraindicated in a patient with a history of breast cancer. D. Black cohosh is an herbal remedy marketed for use with hot flashes. However, it has not been demonstrated to be more effective than a placebo for this use. In addition, it has been posited to have a possible estrogenic effect on breast tissue, which should be avoided in patients with a history of breast cancer. Vital Concepts: Selective serotonin reuptake inhibitors (SSRI)s are the most effective pharmacologic choice for the vasomotor symptoms of menopause. Citalopram is an SSRI that has good evidence of efficacy against these symptoms and is typically well-tolerated.

What would you expect to find on a wet-mount slide of a patient with bacterial vaginosis (BV)? A. Epithelial cells and RBCs B. Tzanck cells C. Clue cells D. Leukocytes and epithelial cells

Correct Answer: C. Clue cells Clue cells would be found in the wet mount of a person diagnosed with BV. Incorrect Answers: A. This would not be found in a BV diagnosis.B. The Tzanck cell is associated with varicella not BV. D. This would not be found in a BV diagnosis.

An otherwise healthy 35-year-old female presents with a persistent annoying cough of two weeks' duration. She states that she initially had a temperature of about 99 degrees Fahrenheit with a runny nose. At that time, her cough was mild, but now she states that she has been experiencing paroxysm of coughing frequently without any apparent reason. On several occasions, she has vomited afterwards. Today, she is afebrile and her lungs are clear. Which of the following is most likely to be true in this case? A. Td should be given B. CT chest is indicated C. Contact precautions should include respiratory droplet precautions D. Levofloxacin is indicated as first-line therapy for this patient

Correct Answer: C. Contact precautions should include respiratory droplet precautions The patient in this case has a presentation that is typical of pertussis, or whooping cough. This disease is caused by the gram negative bacteria Bordetella pertussis. Pertussis is a coughing illness of at least 2 weeks' duration with one of the following findings: • Inspiratory whooping or post-tussive vomiting without apparent cause • Paroxysmal coughingThere are three stages of pertussis:• Catarrhal stage: low-grade fever, rhinorrhea, mild cough • Paroxysmal stage: Severe cough with inspiratory "whooping" sound and post-tussive vomiting in some cases • Convalescent stage: cough becomes less frequent and resolves Laboratory investigations include use of a nasopharyngeal swab for culture and polymerase chain reaction; pertussis antibodies by enzyme-linked immunosorbent assay (ELISA); and CBC (increased WBC count with lymphocytosis up to 80%). Chest radiographs are normally negative, unless a secondary bacterial infection is present. First-line treatment is a macrolide antibiotic. Chemoprophylaxis is recommended for close contacts, with respiratory droplet precautions. Antitussives, mucolytics, hydration, and rest are recommended. Patients who require a booster vaccination against pertussis should receive the Tdap instead of Td.

An 86-year-old woman with a history of atrial fibrillation presents with 3 days of nasal congestion, fatigue, and a right frontal headache. Her expectorated sputum is cloudy and green-yellow tinged. The patient's past medical history is significant for paroxysmal atrial fibrillation and hypertension. Her medications include warfarin 2.5 mg per day, digoxin 0.25 mg per day, and hydrochlorothiazide 12.5 mg per day. The patient's temperature is 99.3° Fahrenheit (37.4° Celsius), and she has a blood pressure of 138/88 and a regular pulse of 72 bpm. She has a normal white blood cell count, and her international normalized ratio (INR) is within the target range. A sputum culture is sent to the lab, and the patient is sent home with a prescription for Bactrim DS (trimethoprim/sulfamethoxazole 160 mg/800 mg) BID for 10 days pending the culture results. She returns to the clinic 2 days later complaining of nausea, abdominal pain, and diarrhea. She appears drowsy, disheveled, and confused. Her temperature is now 98.6° Fahrenheit (37.0° Celsius), blood pressure is 120/60, and pulse is regular at 48 bpm. What is the most likely cause of this patient's nausea, abdominal pain, and diarrhea? A. Clostridium difficile colitis B. Viral gastroenteritis C. Digoxin toxicity D. Adverse reaction to the antibiotic

Correct Answer: C. Digoxin toxicity Digoxin has a narrow therapeutic range. The decreased renal clearance of digoxin may lead to an increased risk of digoxin toxicity. The American Geriatrics Society (AGS) Beers Criteria for Potentially Inappropriate Medication Use in Older Adults recommends the dosage of digoxin should not exceed 0.125 mg per day in elderly patients for the treatment of atrial fibrillation or heart failure. This patient is taking 0.25 mg of digoxin per day, putting her at risk for digoxin toxicity. The concomitant use of trimethoprim with digoxin has been shown to increase digoxin levels in some elderly patients. Both trimethoprim and digoxin undergo tubular secretion, and the addition of trimethoprim can interfere with the renal tubular secretion of digoxin if used concurrently. This interference reduces the renal clearance of digoxin, causing elevated digoxin levels in the blood. Elderly patients are particularly at risk and should be monitored carefully for digoxin toxicity if trimethoprim is added. This patient was prescribed Bactrim DS (trimethoprim/sulfamethoxazole 160 mg/800 mg) BID. She returns 2 days later with abdominal pain, nausea, and diarrhea and is drowsy and confused with a low heart rate. The typical manifestations of digoxin toxicity consist of gastrointestinal and neurological symptoms and cardiac dysrhythmia, including bradycardia. The most likely cause of this patient's symptoms is digoxin toxicity based on her higher-than-recommended dose of digoxin, the addition of a trimethoprim-containing

A 70 year old male with a history of chronic alcohol use disorder is evaluated by a nurse practitioner. The nurse practitioner notes the patient has a macrocytic anemia. Which of the following is a cause of macrocytic anemia? A. Rheumatoid arthritis B. Chronic autoimmune disorders C. Folate deficiency D. Systemic lupus erythematosus

Correct Answer: C. Folate deficiency Folate deficiency results in macrocytic anemia. Alcoholics are at increased risk of folate deficiency because of poor dietary habits and intestinal malabsorption. Folate is required for maturation of cells and folate deficiency results in defective erythropoiesis. Incorrect Answers: A. Rheumatoid arthritis. Rheumatoid arthritis can be associated with several types of anemia. However, with the provided history of a chronic alcohol use disorder, folate deficiency is the best answer. B. Chronic autoimmune disorders. Chronic autoimmune disorders can be associated with several types of anemia. However, with the provided history of a chronic alcohol use disorder, folate deficiency is the best answer. D. Systemic lupus erythematosus. Systemic lupus erythematosus can be associated with several types of anemia. However, with the provided history of a chronic alcohol use disorder, folate deficiency is the best answer.

A 12-year-old male patient present for evaluation of a painful pimple on his eyelid. On exam, you find a 3 mm fluctuant pustule on the margin his right eyelid with no exudate. What is this likely to be? A. Chalazion B. Molluscum contagiosum C. Hordeolum D. Blepharitis

Correct Answer: C. Hordeolum A hordeolum is also known as a stye. It is a painful infection of the meibomian gland, so located at the margin of the eyelid. It is treated with warm compresses for 10 minutes TID and erythromycin ointment may also be added. Incorrect Answers: A. A chalazion is a subcutaneous nodule that results from blockage of the meibomian gland, but are generally not due to infection. May be associated with a hordeolum. B. Molluscum contagiosum are small nodules with a central umbilication that may appear on the face or eyelids. They are caused by a virus and should not be painful. D. Blepharitis is inflammation of the eyelid margin. Vital Concepts: A hordeolum is also known as a stye. It is a painful infection of the meibomian gland, so located at the margin of the eyelid. It is treated with warm compresses for 10 minutes TID and erythromycin ointment may also be added.

A patient complains of intolerable itching in the pubic area. On examination, the nurse practitioner notes erythematous papules and tiny white specks in the pubic hair. Which of the following would you not expect to present as above? A. Pediculosis pubis B. Scabies C. Impetigo D. Atopic dermatitis

Correct Answer: C. Impetigo While pruritis can be associated with impetigo, it is not typically the presenting symptom. Non-bullous impetigo is the most common form of impetigo and it is characterized by lesions that begin as papules and progress to form vesicles surrounded by erythema. The vesicular lesions become pustular and then breakdown to form a crust that has a characteristic honey colored appearance. Lesions usually involve face and extremities. Bullous impetigo is less common and is characterized by enlargement of the vesicular lesions to form bullae filled with clear yellow fluid. Incorrect Answers: A, B, D. Intense itching is characteristic of pediculosis pubis, scabies, and atopic dermatitis. All of these conditions may present similarly to the patient above and should be included in the differential diagnosis. Vital Concepts: Non-bullous impetigo is the most common form of impetigo and it is characterized by lesions that begin as papules and progress to form vesicles surrounded by erythema. The vesicular lesions become pustular and then breakdown to form a crust that has a characteristic honey-colored appearance. Lesions usually involve face and extremities. Bullous impetigo is less common and is characterized by enlargement of the vesicular lesions to form bullae filled with clear yellow fluid.

The NP knows that the appropriate test to screen for color blindness in an 8 year old is the: A. Cover/uncover test B. Red reflex exam C. Ishihara chart D. Snellen chart

Correct Answer: C. Ishihara chart The Ishihara chart is the appropriate test to use for screening for color blindness. Incorrect Answers: A. The cover/uncover test is a test for ocular deviation to test for strabismus. B. The red reflex exam is used to check the child's pupillary health and to screen for abnormalities at the back of the eye. D. The Snellen chart is used to measure the child's visual acuity.

A 16-year-old male presents because he is concerned about acne. He first began to notice worsening acne 6 months ago and began using over-the-counter benzoyl peroxide and, after 3 months, saw his primary care physician who prescribed tretinoin and oral tetracycline. He states that he has been using both medications for the last 3 months but there is no improvement in acne. He also states he has noticed he is very quick to anger and moody over the last few months and wonders whether that is a side effect of the acne medications. He is a football player for his high school and has been participating in extra practices to "bulk up" in the hopes of achieving a college scholarship. Upon closer inspection, he has gained 30 pounds in the last 3 months but does not appear overweight, rather has increased muscle mass. He states that a friend on the football team has been giving him pills to help his performance. Which of the following is most likely to be identified? A. Increased serum luteinizing hormone levels B. Decreased prothrombin time C. Lowered thyroxine binding globulin (TBG) levels D. Increased serum HDL cholesterol

Correct Answer: C. Lowered thyroxine binding globulin (TBG) levels Anabolic steroids can lower thyroxine-binding globulin (TBG) levels which, in turn, can decrease total T4 levels, although free T4 remains the same. Incorrect Answers: A. This patient has multiple features that are consistent with anabolic steroid use, including acne, a rapid increase in weight and muscle mass, and mood swings. In men steroid use decreases serum luteinizing hormone and follicle-stimulating hormone levels, which leads to decreased endogenous testosterone production, decreased spermatogenesis, and testicular atrophy. There is also an increased estrogen production due to the aromatization of circulating testosterone which can lead to gynecomastia. B. Anabolic steroid use can lead to increased prothrombin time as well as a suppression of clotting factors II, V, VII, and X. D. Anabolic steroid use can have multiple cardiovascular effects including decreased serum HDL cholesterol, increased serum LDL cholesterol, increased total cholesterol, and increased blood pressure. These effects appear to be reversible when anabolic steroid use is discontinued. The risk of sudden death from cardiovascular causes can occur in an athlete abusing anabolic steroids even when there is no evidence of atherosclerosis. This is due to an increased risk of thrombus formation while using anabolic steroids.

Which of the following oral medications is the safest to use during pregnancy? A. Lisinopril B. Losartan C. Methyldopa D. Amlodipine

Correct Answer: C. Methyldopa This concept is essential for all practitioners who will care for pregnant patients, as many antihypertensive medications are teratogenic. Incorrect Answers: (A) (B) Both ACE inhibitors and angiotensin receptor blockers can cause organ malformations, especially renal. They are classified as pregnancy category D. (D) Calcium channel blockers such as nifedipine have little data concerning safety in pregnant patients. Amlodipine in particular has little data in pregnancy and is category C.

The nurse practitioner understands that chronic synovitis with pannus formation is the basic pathophysiologic finding in clients with: A. Systemic lupus erythematosus (SLE) B. Ankylosing spondylitis (AS) C. RA D. Osteoarthritis

Correct Answer: C. RA The chronic inflammatory disorder of RA is associated with synovial hypertrophy from chronic synovitis and pannus formation that results in the progressive destruction of cartilage. Incorrect Answers: A. SLE has a distribution of symptoms similar to that of RA but no pannus formation.B. As usually involves the large peripheral joints and is characterized by extreme kyphosis. D. There is no inflammation with osteoarthritis.

A 72-year-old female complains of sudden onset of a severe headache behind the right eye, with eye pain and tearing. She says she sees halos around the light fixtures in the room. She complains of nausea and vomiting. Examination reveals a blood pressure of 130/90 mm Hg. She is afebrile. Her right pupil is fixed and mid-dilated and appears cloudy. She has conjunctival injection and excessive tearing. Which of the following should the NP do? A. Stain the eye with fluorescein B. Dilate the eye with an anticholinergic drop C. Send the patient to the ED D. Apply antibiotic eye drops

Correct Answer: C. Send the patient to the ED The patient's presentation is consistent with acute angle closure glaucoma. This is the result of sudden blockage of the aqueous humor, resulting in a marked increase in the intraocular pressure. If not treated immediately, it can result in ischemia and permanent damage to the optic nerve (CN II). Patients with acute angle closure glaucoma will typically complain of sudden onset of a severe frontal headache and eye pain, with blurred vision and tearing. They may also experience associated nausea and vomiting. The affected eye will have a fixed and mid-dilated cloudy pupil that looks more oval than round in shape. Conjunctival injection with increased tearing is common. Incorrect Answers: A. Stain the eye with fluorescein. Fluorescein eye stain is used to detect corneal injuries or foreign bodies in the eye. Is not utilized in acute angle closure glaucoma. B. Dilate the eye with an anticholinergic drop. Dilating the pupil with eye drops is utilized during an eye examination to exam the retina, macula and optic nerve. Dilating the eye with anticholinergic drops will worsen angle closure glaucoma and raise intraocular pressure. D. Apply antibiotic eye drops. Bacterial conjunctivitis can be treated with antibiotic eye drops. Bacterial conjunctivitis symptoms include redness and purulent discharge in one eye, sometimes bilateral.

A mother brings her 2-week-old infant for a well-child check. She is primarily breastfeeding him, with occasional formula supplementation. How should you advise her regarding vitamin D intake for her baby? A. Breastfed infants do not need supplemental vitamin D. B. Intake of vitamin D in excess of 200 IU/day is potentially toxic. C. The infant should be given 400 IU of supplemental vitamin D daily. D. Vitamin D supplementation should not be started until the infant is at least 6 months old

Correct Answer: C. The infant should be given 400 IU of supplemental vitamin D daily. In 2008, the American Academy of Pediatrics increased the recommended daily intake of vitamin D in infants, children, and adolescents to 400 IU/day. The reason that breastfed infants are deficient in vitamin D is because mothers who are deficient in vitamin D have vitamin D-deficient milk, which leads to vitamin D deficiency in the infant. Studies are being performed to determine if higher maternal vitamin D supplementation doses can effectively increase breast milk vitamin D levels. In this way, both mother and infant could have sufficient levels of vitamin D, and supplementation would not be necessary. Until these studies are completed, a safe alternative is for infants to receive a vitamin D-only supplement to provide 400 IU vitamin D daily. Incorrect Answers: A. Breastfeeding does not provide adequate levels of vitamin D. Exclusive formula feeding probably provides adequate levels of vitamin D, but infants who consume <1 L of formula per day need supplementation with 400 IU of vitamin D daily. B. 200 IU of daily vitamin D is not an adequate amount for exclusively breastfed infants. D. Vitamin D supplementation should be started within the first 2 months after birth.

A 72-year-old male presents with a chronic cough that produces white sputum. The patient states that he becomes short of breath with exertion. This symptom has become so severe that he was forced to stop smoking. He is also currently being treated for benign prostatic hypertrophy (BPH). Which of the following medications may improve his shortness of breath, but could worsen his BPH related symptoms? A. Oxygen 2-3L via nasal cannula B. Salmeterol diskus C. Tiotropium diskus D. Medrol dose pack

Correct Answer: C. Tiotropium diskus Tiotropium (Spiriva) is an anticholinergic and may be used to manage chronic lung disease, e.g., COPD, but can worsen symptoms of BPH. Anticholinergics can also worsen symptoms of narrow-angle glaucoma. Incorrect Answers: A. Supplemental oxygen therapy does not worsen symptoms of BPH. It is the only treatment known to prolong life in COPD patients. B. Salmeterol diskus does not worsen symptoms in patients with concurrent COPD and BPH. D. A Medrol dose pack does not worsen symptoms of BPH

The NP is caring for a child with Type 1 diabetes. His mother reports a recent history of urinary frequency. The child denies flank pain. Vital signs are stable and the child is not febrile. The NP orders a urinalysis which shows a large amount of ketones and leukocytes and a trace of protein. The most appropriate test to order is: A. 24-hour urine for protein and creatinine clearance B. 24-hour urine for microalbumin C. Urine for culture and sensitivity (C&S) D. An intravenous pyelogram (IVP)

Correct Answer: C. Urine for culture and sensitivity (C&S) Type 1 diabetes predisposes children to urinary tract infections. The urinalysis shows that this is the likely diagnosis. A urine for culture will provide definitive diagnosis and will indicate which antibiotic(s) will cure the infection. Incorrect Answers: A. Creatinine is a product of muscle metabolism and is an indirect measurement of glomerular filtration rate. It is NOT a test for infection. B. The 24-hour urine for microalbumin is a test used to measure albumin in the urine. It is used to screen for early renal disease in diabetics, but is NOT a test for infection. D. An IVP is used to radiologically visualize the kidneys, ureters and urethra. It is not the test of choice for infection.

A 30-year-old female with no significant past medical history presents with vaginal irritation and discharge for 2 days. She says she is 7 weeks pregnant and was recently treated for a UTI with nitrofurantoin for 1 week. She has no dysuria, abdominal pain, or vaginal bleeding. Her vitals are as follows: blood pressure 110/70, heart rate 75, respiratory rate 16, temperature 98.5, and O2 saturation 99% on RA. On exam, a whitish vaginal discharge is noted that is not malodorous. Her abdomen is nontender, she has no cervical motion tenderness, and her os is closed. How should this patient be treated? A. Presumptively treat for PID B. Check vaginal pH; if normal, treat for candidiasis with 1 dose of oral fluconazole C. Presumptively treat for BV D. Check vaginal pH; if normal, treat for candidiasis with miconazole cream applied vaginally for 1 week

Correct Answer: Check vaginal pH; if normal, treat for candidiasis with miconazole cream applied D. vaginally for 1 week This is a yeast (candidiasis) infection. If there is any doubt about the diagnosis, a quick vaginal pH check will reveal a normal pH in a yeast infection. Incorrect Answers: (A) This is not likely PID, as she has no cervical motion tenderness or abdominal pain. Additionally, PID is rare in pregnancy. (B) In a known pregnancy, fluconazole is not recommended due to potential effects on the embryo, so a topical imidazole (either clotrimazole or miconazole) applied vaginally for 7 days is preferred. (C) BV would involve discharge with a foul odor that is thin or gray/brown looking, and the pH would be >4.5 (often ≥5.5).

A 20-year-old presents to the office for a wellness check. What is the prevalence of bulimia nervosa in female college students? A. 1% B.2% C. 4% D. 5%

Correct Answer: D. 5% Bulimia nervosa occurs in approximately 5% of female college students, who are 10x more likely than male college students to develop this disorder. The prevalence of anorexia nervosa is 1.5% in teenage girls overall. Incorrect Answers: (A), (B), (C) are the not correct prevalence.

A 17-year-old male with no PMH presents with shoulder pain that has occurred for several weeks. He states that he has been playing basketball for years and recently injured his shoulder when he dove for a loose ball and fell on his outstretched hand. In the few days after the injury, his shoulder was stiff and swollen. However, over time, the shoulder pain has concentrated to a specific region on the lateral portion of his clavicle that worsens in the evening. His shoulder is stiff and painful, especially when he wakes up. He also feels a "popping and locking" of the shoulder joint. Pain is not related to temperature. The patient's temperature = 98°F, HR = 70, RR = 14, and BP = 110/75. Physical exam shows pinpoint tenderness over the lateral portion of the clavicle. Which of the following is the initial diagnostic test to evaluate his condition? A. US B. MRI C. Stress X-rays studies D. AP plain film

Correct Answer: D. AP plain film An acromioclavicular sprain is a tearing of the acromioclavicular and/or coracoclavicular ligament at the outside end of the collarbone near the shoulder. It is caused by a strong, direct blow to the front or top of the shoulder from trauma or by a fall on an outstretched hand. After the initial trauma, patients experience generalized shoulder tenderness and swelling. However, over time, the athlete will complain of specific tenderness at the acromioclavicular joint that is worse in the evening and a stiff and sore shoulder when awakening. Pain is exacerbated with lifting exercises such as weight training. Popping of the AC joint may occur as well. The combination of trauma to his shoulder from falling on an outstretched arm, tenderness at the AC joint, and worsening of symptoms during the evening confirm the diagnosis of AC sprain. An AP radiograph of the shoulder can be used to classify the injury. Stress radiographs are no longer used routinely. Incorrect Answers: (A), (B), (C) US, MRI and stress radiographs are not indicated for acromioclavicular sprains.

A 42 year-old otherwise healthy patient has had yellowish green nasal discharge and frontal headache for over two weeks. The client's temperature has gone up to 101.2 degrees Fahrenheit on most afternoons. She has a cough that worsens when she lies down. The findings on physical exam are within normal limits except for the drainage and a slightly erythematous pharynx. She does not have any drug allergies and has not been taking any medications in the last few months. Which of the following medications would be most appropriate to prescribe for her? A. Diphenhydramine B. Erythromycin C. Pseudoephedrine D. Amoxicillin with clavulanate

Correct Answer: D. Amoxicillin with clavulanate The client is experiencing symptoms of moderately severe acute sinusitis as indicated by her symptoms of facial pressure, headache, and postnasal discharge. The first line antibiotic is amoxicillin/clavulanate because of its safety and efficacy. Incorrect Answers: A. Oral antihistamines such as diphenhydramine should not be used unless the client has allergies. B. Erythromycin is not a first-line antibiotic for acute sinusitis. C. Oral decongestants (like pseudoephedrine) are not as effective as topical agents (for example, nasal steroid sprays) for clients with sinusitis

Which of the anti-hypertensive agents listed below is contraindicated in pregnancy? A. Beta-blockers B. Triamterene-containing diuretic C. Central alpha agonists D. Angiotensin II receptor blockers

Correct Answer: D. Angiotensin II receptor blockers Angiotensin II receptor blockers are pregnancy class D drugs and therefore contraindicated in pregnancy. The malformations are related to the sensitivity of the fetus to the hypotensive action of the drug. Incorrect Answers: A. In a 2003 systematic review of 13 population-based case-control or cohort studies examining the risk of congenital malformations associated with first-trimester oral beta-blocker exposure compared with no exposure, there was no overall increase in major congenital malformations. Labetalol has both alpha- and beta-adrenergic blocking activity and may preserve uteroplacental blood flow to a greater extent than traditional beta-blockers. B. Triamterene is Pregnancy Risk Factor Class C and is not contraindicated in pregnancy. C. Central alpha agonists are not contraindicated during pregnancy. Methyldopa is Pregnancy Risk Factor Class B/C.

A 20-year-old female soccer player was chasing another player and felt a pop in her left knee as she was cutting. She had significant swelling and difficulty bearing weight on her left leg, believing that her knee was going to give out. The patient's temperature is 98.3°F, HR is 70, RR is 16, and BP is 120/80. Her BMI is 28. Physical exam reveals tenderness over the anterior knee portion and swelling all around the knee. You suspect an ACL tear and want to perform an examination to assess knee damage. Which of the following tests is considered to be the least helpful in diagnosing an acute ACL tear of the knee? A. Lachman test B. Pivot-shift test C. MRI of the knee D. Anterior drawer test

Correct Answer: D. Anterior drawer test The anterior drawer test involves the patient lying supine with the hips flexed to 45 degrees; the knee is flexed to 90 degrees, and the feet are flat on the table. The examiner grasps the tibia just below the joint line of the knee. The thumbs are placed along the joint line on either side of the patellar tendon. Then, the tibia is drawn forward anteriorly. If the tibia pulls forward more than normal, the test is considered positive for an ACL tear. This test has the lowest positive predictive value (PPV) of all of the ACL tests at 29%. The anterior drawer has good sensitivity and specificity in chronic conditions (both >90%), but it is not typically accurate in acute injury. Incorrect Answers: A. In the Lachman test, the knee is flexed at 20-30 degrees with the patient supine. The examiner placesa hand behind the tibia, and the other grasps the patient's thigh. The tibia is pulled forward to assess the amount of anterior motion of the tibia in comparison to the femur. More than about 2 mm of anterior translation compared to the uninvolved knee suggests a torn ACL, as does 10 mm of total anterior translation. This test has better PPV than the anterior drawer test at 59%. B. The pivot-shift test is performed while the patient is lying supine with the hip flexed to 30 degrees. The lower leg and ankle are grasped while maintaining 20 degrees of internal tibial rotation. The opposite hand grasps the lateral portion of the leg at the level of the superior tibiofibular joint. While maintaining internal rotation, a valgus force is applied to the knee while it is flexed slowly. If the tibia's position on the femur reduces as the knee is flexed 30-40 degrees, or if there is an anterior subluxation felt during extension, the test is positive for an ACL tear. This test also has a higher PPV than the anterior drawer test at 69%.

A nurse practitioner is reviewing a patient's chart before performing an examination. The previous provider noted that the patient had a normal Rinne test. Which of the following cranial nerves did the provider assess? A. CN V B. CN VI C. CN VII D. CN VIII

Correct Answer: D. CN VIII The Rinne test evaluates conductive hearing loss. The test is performed by striking a tuning fork of 256 or 512 Hz and placing it on the mastoid process. A normal test will show air conduction time that is twice as long as bone conduction time. if there is hearing loss bone conduction is longer than air conduction. CN VIII is the auditory nerve. Incorrect Answers: A. CN V. The trigeminal nerve (CN V) is responsible for facial movement and sensation. To test CN V check pinprick sensation of bilateral face. Also have patient open jaw against resistant. B. CN VI. The abducens nerve (CN VI) is responsible for movement of the lateral rectus eye muscle. To test CN VI test extraocular eye movements. C. CN VII. The facial nerve (CN VII) is responsible for muscles of facial expression and taste. To test CN VII ask the patient to smile and look for any hemifacial weakness.

A researcher is studying potential risk factors for obesity. She sends a questionnaire to 5000 patients who meet the CDC criteria for obesity that asks them about sedentary lifestyle factors, consumption of processed food, and family history of obesity. The researcher sends the same questionnaire to 5000 patients who are not obese. Which of the following best describes the design of the study? A. Cross-sectional study B. Randomized controlled trial C. Meta-analysis D. Case-control study

Correct Answer: D. Case-control study This is a retrospective case-control study. In this observational study design, subjects are selected based on an outcome, and their prior exposures are assessed and compared to subjects who do not have the particular outcome. The result of the study is expressed as an odds ratio: OR = (# exposed with disease / # exposed without disease) / (# not exposed with disease / # not exposed without disease). A retrospective case-control study does not prove causation but demonstrates an association. Incorrect Answers: A. A cross-sectional study is used to assess disease status and risk factors at 1 point in time. B. A randomized controlled trial refers to a study where a group is given a treatment and outcomes are compared against a control group given a placebo or treated with the standard of care. C. A meta-analysis provides a statistical analysis of the results of multiple studies. Vital Concepts: In an observational study design, subjects are selected based on an outcome, and their prior exposures are assessed and compared to subjects who do not have a particular outcome. The result of the study is expressed as an odds ratio: OR = (# exposed with disease / # exposed without disease) / (# not exposed with disease / # not exposed without disease). A retrospective case-control study does not prove causation but demonstrates an association.

A female renal transplant patient is on cyclosporine. Which common side effect should she be warned about? A. Thrombocytopenia B. Neutropenia C. Myopathy D. Gingival hyperplasia

Correct Answer: D. Gingival hyperplasia Gingival hyperplasia is a common side effect of cyclosporine. Other effects include nephrotoxicity, hypertension, hirsutism, tremors, dysesthesias, hepatotoxicity, and seizures. Incorrect Answers: A. Thrombocytopenia is a rare but serious side effect and is also toxicity due to ATG (polyclonal). B. Neutropenia is a rare side effect and also occurs with azathioprine.C. Myopathy may result from steroid therapy.

While providing care to children in Bangladesh you notice that many of the children present with the same complaint. On further discussion, you learn that they eat a diet of grains, dairy and occasional meat with no fruits and vegetables. What is the most likely chief complaint? A. Night blindness B. Diarrhea, dermatitis, and dementia C. Megaloblastic anemia D. Irritability, bleeding gums, and periosteal hemorrhage

Correct Answer: D. Irritability, bleeding gums, and periosteal hemorrhage Infantile scurvy presents with irritability, bone tenderness, bone swelling, and pseudoparalysis of the legs.Scurvy may occur in infants who are fed unsupplemented cow's milk in the first year of life or if the diet islacking fruits and vegetables. In older individuals, scurvy presents with osmotic diarrhea, bleeding gums,perifollicular hemorrhage, arthropathy, and painful bones. Although scurvy is rare in children, it is still reported in children who ingest only well-cooked foods and few fruits or vegetables. Dietary sources of vitamin C include papaya, citrus fruits, cantaloupe, strawberries, tomatoes, cabbage, and potatoes. Incorrect Answers: A. A diet consisting of grains, dairy and occasional meat is lacking in vitamin C and could lead to scurvy, which typically presents with irritability and bleeding gums. Night blindness is a symptom of vitamin A deficiency. Vitamin A deficiency is the leading cause of preventable blindness in children in many countries, especially developing countries in Africa and Southeast Asia. Vitamin A deficiency also increases the risk of severe illness and death from common childhood infections, including measles. There are 3 types of vitamin A: carotenoids, beta-carotene and retinols (found in animal sources of food). Breastmilk contains adequate vitamin A and vitamin A is also found in liver, chicken, beef, eggs, leafy green vegetables, carrots, and sweet potatoes. B. Pellagra, a niacin deficiency, is characterized by diarrhea, dermatitis, and dementia (the 3 D's). Gastrointestinal symptoms associated with niacin deficiency other than diarrhea include glossitis, angular stomatitis, and cheilitis. The dermatitis of pellagra is characteristic and begins as painful erythema in areas of sun-exposed skin which can progress to an exudative phase with vesicle or bullae formation. Neuropsychiatric symptoms include insomnia, fatigue, nervousness, irritability, apathy, memory impairment and depression and, when left untreated, may progress to dementia. Pellagra can result in death (the 4th D) if left untreated. Niacin is found in animal protein, beans, and fortified cereals and can be synthesized in the

A 54-year-old male arrives at the office for a wellness check. He is on an MAOI medication. Which of the following foods does NOT have to be avoided by this patient taking an MAOI? A. Pickled herring B. Smoked salmon C. Chianti wine D. Lima beans

Correct Answer: D. Lima beans Lima beans are a safe food to eat while taking MAOI medications.Incorrect Answers: C A. Foods to be avoided include cheese other than cottage and cream cheese, caviar, liver, ✔D smoked/pickled/cured fish and meat, ripe avocados, canned figs, yeast extract, chianti red wine, and beerwith yeast. B. Foods to be avoided include cheese other than cottage and cream cheese, caviar, liver, smoked/pickled/cured fish and meat, ripe avocados, canned figs, yeast extract, chianti red wine, and beer with yeast. C. Foods to be avoided include cheese other than cottage and cream cheese, caviar, liver, smoked/pickled/cured fish and meat, ripe avocados, canned figs, yeast extract, chianti red wine, and beer with yeast. Vital Concept: The practitioner must be aware of prescribing guidelines. MAOI inhibitors should be avoided with cheese other than cottage and cream cheese, caviar, liver, smoked/pickled/cured fish and meat, ripe avocados, canned figs, yeast extract, chianti red wine, and beer with yeast. Foods in moderation include chocolate and coffee. Avoid cold meds, allergy meds, decongestants, cough meds, stimulants, meperidine, SSRI, bupropion, mirtazapine, nefazodone, trazodone, and venlafaxine.

A 24-year-old tennis player complains of knee pain upon presentation to the orthopedic clinic. She claims that during one of her games a month ago, she quickly changed direction in order to return a served ball from her opponent. She felt an uncomfortable painful twisting motion as she reached for the ball. After the incident, she was able to put weight on her knee and felt fine walking. However, recently she has felt increasing knee pain when walking, joint-line tenderness, and difficulty putting weight on her knee. The patient's temperature = 98.3°F, HR = 70, RR = 16, and BP = 120/80. Her BMI is 24. A McMurray test is performed: the patient feels pain in the knee when it is flexed completely and the examiner provides a valgus stress. Which of the following is the most likely diagnosis? A. ACL tear B. MCL tear C. LCL tear D. Medial meniscus tear

Correct Answer: D. Medial meniscus tear Meniscus tears occur when the knee is twisted while flexed with the foot planted on the ground. Unlike ACL, MCL, or LCL tears, there is typically no trauma involved and no immediate pain following the injury. Patients can walk and bear weight on the knee following the injury, In the days that follow, they present with symptoms of joint-line tenderness, difficulty squatting, joint effusion, and loss of smooth motion of the knee. Meniscus tears are diagnosed by the McMurray test: the knee is held by one hand, which is placed along the joint line, and the knee is flexed completely while the foot is held by the sole with the other hand. The examiner then places one hand on the lateral side of the knee to stabilize the joint and provides a valgus stress. The other hand rotates the leg externally while extending the knee. If pain is felt, this indicates a tear in the medial meniscus. Our patient's pain 1 month after the incident followed by a positive valgus stress on McMurray test confirm the diagnosis of a medial meniscus tear. Incorrect Answers: A. Tearing of the anterior cruciate ligament (ACL) presents with immediate pain at the moment of impact from either a direct blunt trauma to the knee or hard planting of the knee on the ground when changing directions, especially in sports such as basketball, football, or soccer. Patients present with immediate severe pain, swelling, and sometimes hemarthrosis. Patients also have difficulty in putting weight on their knees and patellar tenderness. Diagnosis is made using a variety of tests such as the Lachman, anterior drawer, or pivot tests. The lack of immediate pain following the incident make this diagnosis unlikely. B. Tearing of the medial collateral ligament (MCL) typically occurs when a direct, blunt trauma is dealt to the lateral portion of the knee, causing the knee to bend inwards and tearing the MCL. Unlike ACL tears, there is no swelling or ecchymosis found in MCL tears. Patients present with difficulty in bearing weight and with

The McMurray's test is performed by having the patient lie supine, flexing the knee and hip to 45 degree angles. While one hand holds the knee, the other holds the ankle. Then externally rotate the leg and assess for any audible clicks or pain. This is testing for: A. Tenosynovitis B. Knee osteophytes C. Arthritis D. Meniscal injury Correct

Correct Answer: D. Meniscal injury A positive McMurray's test indicates a meniscal injury. Incorrect Answers: A. Tenosynovitis would not have a positive McMurray's test. Tenosynovitis is inflammation of the sheath surrounding a tendon. You would use Finklestein's test to assess for tenosynovitis. B. Knee osteophytes would not elicit a positive McMurray's test. Osteophytes are suggestive of arthritis. C. This test does not assess for arthritis.

A 32-year-old male presents with a productive cough. He states that this cough started 2 weeks ago and has gotten progressively worse. He denies any fevers, changes in weight, or hemoptysis. His sputum is green in color, and he also complains of a runny nose and muscle soreness. Scattered wheezes are auscultated on physical exam. Which of the following statements are correct regarding his diagnosis? A. Order chest X-ray to rule out pneumonia. B. Order chest X-ray to diagnose bronchitis. C. Order chest X-ray to rule out tuberculosis. D. No radiologic imaging should be performed.

Correct Answer: D. No radiologic imaging should be performed. This patient has acute bronchitis. Acute bronchitis has no clear-cut definition for the duration of symptoms, but the general acceptance is cough for <3 weeks. A productive cough is the predominant symptom. Fever and other systemic symptoms are usually absent. Muscle pains, pleuritic chest pain, fatigue, headache, and runny nose can be present. Care for acute bronchitis is primarily supportive, as therapy is generally focused on alleviation of symptoms. Among otherwise healthy individuals, antibiotics have not demonstrated any consistent benefit in acute bronchitis because the etiology is most likely viral in nature. Chest radiography should be performed in patients whose physical examination findings suggest pneumonia. Elderly patients may have no signs of pneumonia; therefore, chest radiography may be warranted in these patients, even without other clinical signs of infection. No radiographic imaging is used to diagnose bronchitis. Incorrect Answers: A. Chest x-ray is the image of choice used to diagnose pneumonia, but this patients does not exhibit any signs of pneumonia. Pneumonia usually presents with systemic signs such as fever. B. A chest X-ray is not used to diagnose bronchitis. C. This patient does not exhibit any symptoms of tuberculosis such as hemoptysis, night sweats, or fever. If these symptoms were present, then a chest X-ray would be indicated.

You are seeing a 42 year old patient for a physical exam. He has no complaints at this time but on funduscopic exam you note the following image. (See picture below). Which of the following diagnoses do you suspect? Acute angle closure glaucoma B. Diabetic retinopathy C. Retinoblastoma D. Open angle glaucoma

Correct Answer: D. Open angle glaucoma As you can see in the below image of a normal funduscopic exam, the cup to disc ratio should be low. In the patient above, the cup to disc ratio is very high. Since the patient is not having symptoms, it is unlikely to be Angle-Closure Glaucoma. Open angle glaucoma is caused by a slow clogging of the drainage canals in the eye, resulting in increased eye pressure over time. In Acute Angle-closure glaucoma, there are completely blocked drainage canals which results in a sudden rise in intraocular pressure, ocular pain, seeing halos around lights, a red eye, nausea and vomiting, suddenly decreased vision, and a fixed, mid-dilated pupil. Incorrect Answers: (A) Since the patient is not having symptoms, it is unlikely to be Angle-Closure Glaucoma. (B) The patient may have symptoms of floaters, blurriness, dark areas of vision, and difficulty perceiving colors. (C) Symptoms may show a pupil that appears white when light is shined into the eye, sometimes with flash photography.

A 14-year-old female soccer player presents for right knee pain and swelling. She states during a game over the weekend, she injured her knee. She does not recall the mechanism of injury, but her knee gave way and she heard a "pop." She noticed some swelling within an hour and has had pain since. She has no history of a knee injury. She denies any locking but feels instability in her knee. On examination, you notice mild to moderate anterior swelling. She has some physiologic knee valgus deformity. She is tender over the medial patellar edge on palpation. Her range of motion is limited due to the pain and swelling. She has negative varus and valgus stress tests and a negative Lachman's test with good end-point. The patient also has a negative anterior and posterior drawer test and a negative McMurray test. She has some patellar apprehension. Her right knee X-rays show some soft-tissue swelling but no fracture or other abnormalities. What is the most likely diagnosis? A. ACL rupture B. PCL rupture C. Patellar tendon rupture D. Patellar sub-luxation

Correct Answer: D. Patellar sub-luxation Patellar subluxation is the most likely diagnosis in a teenage girl who presents with giving-way episodes of the knee. This injury occurs more often in girls and young women because of an increased quadriceps angle (Q angle), usually greater than 15 degrees. Patellar apprehension is elicited by subluxing the patella laterally, and a mild effusion is usually present. Moderate to severe knee swelling may indicate hemarthrosis, which suggests patellar dislocation with osteochondral fracture and bleeding.

You evaluated a 6-month-old boy with a red discoloration of the left face that has been present since birth. Prenatal, perinatal, postnatal, and family histories were unremarkable. His growth and development were normal. The examination was unremarkable except for a flat, pink-to-red discoloration of the left cheek below the zygoma to the mandible, measuring approximately 3cm by 2cm with well-defined borders. This discoloration did not cross the midline. The color deepened when he cried. What is the most likely diagnosis? A. Capillary hemangioma B. Sturge-Weber syndrome C. Arteriovenous malformation D. Port wine stain

Correct Answer: D. Port wine stain A port-wine stain (nevus flammeus) is a capillary malformation in which dermal capillaries and postcapillary venules are ectatic. Vascular endothelial growth factor may play a role. Most stains are located in the head and neck. Unlike capillary hemangiomas, they tend to be flat and histologically are nonproliferative. They are congenital (only very rarely acquired) but sometimes are not observed at birth (e.g. in the face of severe anemia). They usually begin as pink in color, progressing to red in later childhood and purple into adulthood. They may fade somewhat during the first year but do not disappear entirely. They will grow proportionately with the child. The majority are present on the head and neck. Of those on the face, 45% involve only the distribution of one of the three major branches of the fifth cranial (trigeminal) nerve, while 55% involve more than one branch. Ten percent of those involving the ophthalmic (first) branch of the trigeminal nerve are associated with eye or central nervous system abnormalities. Incorrect Answers: A. Capillary hemangiomas are proliferative, raised lesions that do tend to involute. B. Sturge-Weber syndrome consists of the triad of a facial port wine stain, ipsilateral meningeal capillary malformations, and ipsilateral cerebral cortical capillary malformations, often with seizures and intellectual disability. C. Arteriovenous malformations are abnormal connections between arteries and veins and are usually deeply situated.

A 15-year-old male with no PMH presents with shoulder pain that has occurred for several months. He has been playing baseball for the last 3 years and began feeling pain in his arm and shoulder when he started learning and practicing the breaking-ball pitch over the last 6 months. He denies any trauma. Pain only occurs when the patient is pitching and is relieved by rest. Pain also tends to occur in the deceleration portion of his pitch and is not related to temperature or time of day. The patient's temperature is 98°F, HR is 70, RR is 14, and BP is 110/75. Physical exam shows pinpoint tenderness where the shoulder meets with the arm. What is the most likely diagnosis? A. Thoracic outlet syndrome B. Rotator cuff tendinitis C. Shoulder impingement D. Proximal humerus stress injury (little leaguer's shoulder)

Correct Answer: D. Proximal humerus stress injury (little leaguer's shoulder) Proximal humerus stress injury affects the growth plate of the proximal humerus after repetitive microtrauma. It occurs in young pitchers who throw many breaking balls. Patients present with arm and shoulder pain when throwing the ball that increases in intensity over time. The pain is worsened in the late cocking or deceleration phases of throwing and resolves with rest. Patients complain of point tenderness over the shoulder physis, and pain is reproduced when throwing. This patient has pinpoint tenderness, probably due to microtrauma on the physis from long periods of pitching a breaking ball. These problems, along with relief of shoulder pain at rest, make little leaguer's shoulder most likely. Incorrect Answers: A. Thoracic outlet syndrome occurs when there is compression at the superior thoracic outlet due to excess pressure placed on a neurovascular bundle between the scalene muscles. Because the nerves of the upper limb and/or vessels pass through the outlet, all of them can be affected. Typically, the syndrome is caused by congenital abnormalities (e.g. cervical rib or prolonged transverse process) or by trauma or repetitive strain. Patients present with sharp, burning pain in the upper arm, forearm, and/or fingers. Decreased coloring of the hands, tingling, and coldness are found as well. This patient does not present with any tingling or burning pain, and thus this condition is unlikely. B. Rotator cuff tendinitis involves the swelling of the rotator cuff tendons. This typically is caused by repetitive activities associated with overuse of the arm and shoulder, especially in physical activities such as volleyball or baseball. Symptoms involve shoulder weakness and difficulty lifting the arm over the head. In addition, there may be pain and swelling in the front of the shoulder, a clicking sound when the arm is raised, and stiffness. Resting the shoulder, icing it, and ibuprofen treatment should improve the symptoms. This patient has repetitive trauma and stress but the pinpoint location of his pain makes little leaguer's shoulder

A 65-year-old female presents to the office for a wellness check. What is the best next step to take for a patient with the funduscopic exam in the image below? (add image of papilledema) A. Normal exam, no further steps needed B. Refer to ophthalmology as outpatient C. Patient is likely diabetic, check finger-stick glucose and HBA1c D. Send for urgent neuro evaluation and imaging

Correct Answer: D. Send for urgent neuro evaluation and imaging This is papilledema, which is optic disc edema due to intracranial hypertension. This can be caused by an intracranial mass, cerebral swelling due to infarction or trauma, increased CSF, hydrocephalus, outflow obstruction due to thrombosis or other mass/compression, or pseudotumor cerebri, among other things. All cases of papilledema need to be referred for urgent evaluation by neurology/neurosurgery and head imaging. In contrast, the image of a normal funduscopic exam is also seen below. Note how you can see the sharp clear edges of the disc in the normal eye Incorrect Answers: A. This is not a normal examination.B. This patient requires urgent imaging and neurology consultation. C. Fingerstick and laboratory workup is not the priority in this case. Vital Concepts: All cases of papilledema need to be referred for urgent evaluation by neurology/neurosurgery and head imaging.

A parent presents to your clinic with her four year old daughter who just started pre-kindergarten. She states her daughter has been complaining of headache and sore throat for a couple of days and has had a mild fever. She was just treating it at home until she noticed small blisters in her daughter's mouth and hands. She's concerned her daughter has contracted something from a child at school. You diagnose her child with Hand-foot-mouth disease and prescribe: A. Tamiflu B. Penicillin 500mg three times a day for ten days C. Acyclovir five times per day for ten days D. Treat symptoms with ibuprofen or acetaminophen, cold fluids and salt water gargles

Correct Answer: D. Treat symptoms with ibuprofen or acetaminophen, cold fluids and salt water gargles This is a viral, self-limiting condition that requires only symptomatic treatment. Incorrect Answers: A. This is not influenza. Tamiflu is indicated within 72 hours of influenza onset. B. This is not a streptococcal infection. These symptoms may be confused with streptococcal pharyngitis however, the blisters on the hands would refute this diagnosis. C. This is not varicella. Varicella vesicles are more widespread. Vital Concepts: Hand-foot-mouth disease is a viral, self-limiting condition that requires only symptomatic treatment.

A 45-year-old woman presents with abnormal uterine bleeding. You discuss your recommendation for endometrial sampling to rule out endometrial hyperplasia and carcinoma. What is the most common cause of endometrial cancer? A. Human papilloma virus B. Hereditary nonpolyposis colorectal cancer C. Multiple endocrine neoplasia D. Unopposed estrogen

Correct Answer: D. Unopposed estrogen Unopposed estrogen can lead to overgrowth of endometrial tissue, ultimately prompting endometrial hyperplasia and endometrial cancer. Endometrial cancers occur that are not related to estrogen exposure, which have a different pathologic appearance and a worse prognosis. Women with anovulation or other types of unopposed estrogen exposure should have exposure to progesterone or progestins at least every 3 months to reduce the risk of endometrial hyperplasia and cancer. Incorrect Answers: (A) Human papilloma virus (HPV) is not the most common cause of endometrial cancer. (B) Hereditary nonpolyposis colorectal cancer (HNPCC) is not the most common cause of endometrial cancer. (C) Multiple endocrine neoplasia (MEN) is not the most common cause of endometrial cancer.

Which of the following RBC indices and laboratory markers are consistent with the diagnosis of iron-deficiency anemia? A. Low MCV, low RDW, low reticulocyte count, low iron, low TIBC, low ferritin, low transferrin B. Low MCV, elevated RDW, elevated reticulocyte count, low iron, low TIBC, low ferritin, low transferrin C. Low MCV, elevated RDW, low reticulocyte count, low iron, elevated TIBC, low ferritin, high transferrin D. Elevated MCV, low RDW, elevated reticulocyte count, low iron, elevated TIBC, low ferritin, low transferrin

Correct Answer: Low MCV, elevated RDW, low reticulocyte count, low iron, elevated TIBC, low ferritin, C. high transferrin Iron-deficiency anemia is the most common cause of anemia in children ages 9 to 24 months. It is a microcytic (low MCV, defined as a mean corpuscular volume of <80 μm3 [80 fL]) and hypochromic anemia. It is associated with an elevated RDW, reflecting a wide distribution of RBC sizes (note that RDW is often normal in anemia of chronic disease) and a low reticulocyte count (reflecting a lack of production of new RBCs). Fe studies demonstrate low iron, high TIBC (referring to the ability of unsaturated transferrin to bind iron, which will increase during iron deficiency because more receptors are available), low ferritin (lower iron in the body means that less is stored), and high transferrin (a major transporter of iron, of which the liver increases production in iron deficiency). Risk factors for iron-deficiency anemia include high milk intake, prematurity, blood loss, vegetarian diet, and elevated serum lead. Fe-deficiency anemia and even Fe-deficiency without anemia has adverse effects on attention span, behavior, and school performance. Treatment involves correction of the underlying etiology when possible (e.g. decreased milk intake) and supplemental iron therapy. Incorrect Answers: A. Low RDW, low TIBC, and low transferrin are not consistent with iron-deficiency anemia.B. Elevated reticulocyte count, low TIBC, and low transferrin are not consistent with iron-deficiency anemia. D. Elevated MCV is not consistent with iron-deficiency anemia.

Which medications are most useful when taken during early symptoms of a migraine? (Select all that apply) A. Codeine B. Triptans C. Ergotamines D. Acetaminophen

Correct Answers: B. Triptans C. Ergotamines D. Acetaminophen Treating migraines during the earliest symptoms can be quite effective with triptans. Triptans work by inhibiting the release of vasoactive peptides. They also promote vasoconstriction and block pain pathways in the brainstem. Acetaminophen is also effective if taken early with non-incapacitating attacks. Ergotamines are effective agents, but their use has declined due to the use of triptans. Ergotamines have important contraindications as well as side effects which the provider needs to be aware of before ordering. Incorrect Answer: A. Codeine alone is not a first-line treatment for early migraines.

special considerations for Angiotensin receptor blockers

Do not use in combination with ACE inhibitors or direct renin inhibitor. There is an increased risk of hyperkalemia in CKD or in those on K+ supplements or K+-sparing drugs. There is a risk of acute renal failure in patients with severe bilateral renal artery stenosis. Do not use if patient has history of angioedema with ARBs. Patients with a history of angioedema with an ACE inhibitor can receive an ARB beginning 6 weeks after ACE inhibitor is discontinued. Avoid in pregnant females or females of reproductive age without adequate contraception. Lower risk of cough than ACE-Is.

special considerations for ACE inhibitors

Do not use in combination with ARBs or direct renin inhibitor. There is an increased risk of hyperkalemia, especially in patients with CKD or in those on K+ supplements or K+-sparing drugs. There is a risk of acute renal failure in patients with severe bilateral renal artery stenosis. Do not use if patient has history of angioedema with ACE inhibitors. ACE inhibitor cough is common, in 5 to 20 percent of patients, due to bradykinin production. Avoid in pregnant females or females of reproductive age without adequate contraception

When should you Refer to a Nephrologist or Cardiologist in a hypertensive patient?

-signs of end-organ damage -evidence of a secondary cause of hypertension -only on one to two medications Generally, failure to achieve blood pressure goal in patients who are adhering to full doses of an appropriate three to four drug regimen that includes a diuretic may warrant referral to a nephrologist or cardiologist Before referring, clinicians should first review other causes of inadequate hypertension control such as: -Improper blood pressure measurement -White coat hypertension -Excess sodium intake -Medication issues (e.g., nonsteroidal anti-inflammatory drugs (NSAIDs), illicit drugs, sympathomimetics, oral contraceptives) -Excess alcohol intake -Underlying identifiable causes of hypertension (secondary hypertension)

special considerations for Calcium channel blockers

Associated with dose-related pedal edema, which is more common in females than men.

special considerations for Thiazides

Chlorthalidone is preferred on the basis of prolonged half-life and proven trial reduction of CVD. Monitor for hyponatremia and hypokalemia, monitor uric acid and calcium levels. Use with caution in patients with history of acute gout unless patient is on uric acid-lowering therapy.

A 15-year-old male presents with knee pain which began after kneeing an opponent in a martial arts competition. The pain has not subsided within several weeks, and the patient is not able to participate in his regular activities. You order radiological studies of the left knee, which reveal a bony fragment in the medial condyle of the femur. What is an osteochondritis dissecans (OCD) lesion? A. Bone or cartilage fragment with diminished blood supply from trauma or vascular insult B. Osteophyte (bone spur) growth into cartilage C. Non-union growth plate fracture D. Area of plasma cells in ligaments of spine

Correct Answer: A. Bone or cartilage fragment with diminished blood supply from trauma or vascular insult An osteochondritis dissecans lesion is an area of aseptic necrosis that occurs most commonly in adolescents but can occur in older populations. It often has spontaneous healing, and likely many more people have this condition with self-resolution than is known. Conservative management is rest from activity to avoid repetitive microtrauma to the area. Surgical management is sometimes necessary if the bony fragment is thought to be unstable, as determined by MRI. Incorrect Answers: B. An osteophyte is often the result of a prior insult to the bone or of wear and tear on a joint. C. Premature closure of a growth plate after an injury to the area can occur. A non-union refers to a lack of full healing of a fracture. D. An aggregate of plasma cells would be more consistent with a plasmocytoma.

A 44-year old male with known celiac disease presents with developed vesicles and pustules on his elbows, buttocks, and knees. He does admit to eating gluten while on vacation. He is wondering what treatments are available for his rash. How should you counsel this patient? A. Dapsone will clear the skin lesions in 24-48 hours but will not decrease his risk of gastrointestinal lymphoma. B. Dapsone will clear the skin lesions in 24-48 hours and will decrease his risk of gastrointestinal lymphoma. C. A gluten-free diet will clear the skin lesions in 24-48 hours but will not decrease the risk of gastrointestinal lymphoma. D. A gluten-free diet will clear the skin lesions in 24-48 hours and will decrease the risk of gastrointestinal lymphoma.

Correct Answer: A. Dapsone will clear the skin lesions in 24-48 hours but will not decrease his risk of gastrointestinal lymphoma. This patient has dermatitis herpetiformis, which is a chronic autoimmune blistering condition associated with celiac disease in over 90% of individuals that is more common in men than women. Patients typically present with symmetric, pruritic, pink papules and vesicles on the extensor surfaces (e.g., elbows and knees), buttocks, and scalp, although other sites can also be affected. Often, patients have no intact vesicles on examination but rather superficial erosions or excoriations resulting from scratching. The eruption often flares when patients do not adhere to a strict gluten-free diet, although some patients are unaware they have celiac disease prior to the diagnosis of dermatitis herpetiformis. Patients with dermatitis herpetiformis have an increased risk of other autoimmune conditions such as Hashimoto thyroiditis and of enteropathy-associated T-cell lymphoma. Individuals who follow a strict gluten- free diet will have improved control over their skin disease, although improvement is not immediate. They also have an increased risk of enteropathy-associated lymphoma. Dapsone can lead to rapid clearance of the lesions (24-48 hours) but will not decrease the risk of enteropathy-associated lymphoma. Clinical presentation and skin biopsy can confirm the diagnosis. Blood tests (total serum IgA levels, anti- tissue and anti-epidermal transglutaminase antibodies, and anti-endomysial IgA antibodies) can help confirm the diagnosis. Patients with dermatitis herpetiformis should be referred to gastroenterology to evaluate for GI disease. In addition to these treatments, high-potency topical steroids can help reduce pruritus.

A 55-year-old environmental service worker reports twisting his knee at work yesterday. Upon examination his knee is swollen and tender to palpation. His exam is otherwise negative. He is diagnosed with a Grade II sprain. Your initial treatment plan would include which of the following? A. Protecting and elevating the knee from further injury, use of ice intermittently for 48 hours B. Alternate heat and ice application for 48 hours C. Application of Ace bandage to knee for support D. Isometric exercises, recheck knee in 24 hours

Correct Answer: A. Protecting and elevating the knee from further injury, use of ice intermittently for 48 hours PRICE: Protect, rest, ice, compress and elevate is the acronym used for minor musculoskeletal injuries. Elevation will help decrease swelling to the knee. Application of ice will also minimize tissue swelling if done in the first 48 hours. Incorrect Answers: B. Heat is not recommended in the first 48 hours of an acute musculoskeletal injury. C. An Ace bandage, while helpful, isn't the priority for a Grade II sprain.D. You would avoid any exercises during the acute phase of injury.

To make a diagnosis of allergic rhinitis in the primary care office setting, the nurse practitioner would consider performing: A. A nasal smear B. A total serum IgE C. Skin tests D. RAST

Correct Answer: A. A nasal smear A nasal smear is a simple test and nurse practitioners may obtain. Incorrect Answers: B. Many clients who have uncomplicated allergic rhinitis have a normal serum IgE. C. Skin testing should be performed by trained providers only.D. RASTs cost more and have a lower sensitivity. Vital Concepts: A nasal smear is a simple test and nurse practitioners may obtain to make a diagnosis of allergic rhinitis in the primary care office setting.

The NP suspects that the patient has a Helicobacter pylori infection of the stomach that the NP would like to treat. The patient is already undergoing an upper endoscopy to evaluate persistent GI symptoms. The definitive test for diagnosis of H. pylori is: A. A tissue biopsy from the endoscopy B. An H. pylori titer C. An upper GI series D. A urea breath test

Correct Answer: A. A tissue biopsy from the endoscopy A definitive diagnosis of H. pylori infection is best done through a tissue biopsy from an endoscopy. Incorrect Answers: B. An H. pylori titer only determines if the patient has been exposed to H. pylori during his lifetime. Therefore, the patient may or may not have an active infection. C. Although a clinician might suspect an infection, the upper GI series is an X-ray that will not definitively diagnose H. pylori infection. D. A urea breath test or stool antigen are good non-invasive test that help to diagnose H. pylori infection. However, in patients already undergoing endoscopy, the tissue biopsy continues to be the gold standard for diagnosis of H. pylori infection. In patients not undergoing endoscopy, either a urea breath test or stool antigen would be appropriate.

A nurse practitioner is evaluating a patient with abdominal pain. The patient has a positive iliopsoas sign. Which of the following is the most likely diagnosis? A. Acute appendicitis B. Acute cholecystitis C. Pelvic inflammatory disease D. Acute pyelonephritis

Correct Answer: A. Acute appendicitis The psoas sign is performed with the patient lying on the left side and the examiner hyperextending the patient's right thigh. A positive test results in abdominal pain. This test indicates irritation of the psoas muscle and is considered suggestive of appendicitis.

A 15-year-old female who is 10 weeks pregnant presents with complaints of vaginal bleeding, abdominal pain, vomiting, and muscle spasms for the last 4 hours. Her BP is 165/110. She is febrile, has cold extremities, and is trembling. Abdominal ultrasound confirms a miscarriage with no retained products of conception. Withdrawal from which of the following substances could lead to miscarriage in this female? A. Codeine B. Cannabis C. Cocaine D. Nicotine

Correct Answer: A. Codeine Miscarriage is a complication of opioid withdrawal in the female. During pregnancy, opioid withdrawal, even when treated, can increase the risk of miscarriage and premature birth. In addition, there is a very high rate of relapse after detoxification, and the risk of cycling between intoxication and withdrawal can be even more dangerous to the fetus. Methadone maintenance has generally been accepted as the standard approach to the pregnant woman. Buprenorphine maintenance is also increasingly being used in pregnant opioid- dependent women. Coordinated care between the substance abuse treatment provider and the obstetrical team is of utmost importance. Other complications of IV use of opioids include endocarditis, HIV, HBV/HCV, cellulitis, abscesses, septic arthritis, osteomyelitis, pneumonia, insomnia, myalgias, runny nose, meningitis, pulmonary emboli, and nephrotic syndrome. Incorrect Answers: B. Cannabis use during pregnancy may lead to fetal growth retardation and cognitive and behavioral impairments with psychiatric symptoms in the child. However, withdrawal symptoms in this female indicate opioid abuse. C. Cocaine can lead to fetal hypoxia and placental abruption, but withdrawal symptoms in this female are suggestive of opioid withdrawal. D. These symptoms are not suggestive of nicotine withdrawal.

A 35-year-old man with no PMH is admitted to the primary care clinic with eye problems for the last 5 days. He stated that 8 days ago, he had upper respiratory symptoms, including a runny, stuffy nose, tearing, and throat pain. His symptoms resolved a few days later, but then he started developing problems in his left eye. He states that his left eye has felt itchy, swollen, and red for the last 5 days and has had a persistent watery discharge. He denies any pain to the eye, vision problems, or problems with pupillary reaction. The patient's temperature = 98.3 degrees F, HR = 80, RR = 16, and BP = 110/70. Physical exam of the eye reveals the picture shown below. Which of the following is the next best step in management? A. Cold compresses B. Ayclovir C. Topical azithromycin ointment D. Topical erythromycin ointment and oral azithromycin

Correct Answer: A. Cold compresses Adenovirus is the most common cause of viral conjunctivitis. Typically, this involves an exposure to another individual with the virus, and patients begin complaining of upper respiratory symptoms (such as coughing, congestion, and runny nose) for a few days. Then, the patient notes watery, red, itchy eyes with diffuse hyperemia. The other eye may become affected a few days later. Typically, vision is intact, and pupil size and reaction to light is normal as well. There is rarely pain in the eye or systemic symptoms. The symptoms usually regress within a few weeks, and treatment is supportive, including the use of cold compresses, ocular decongestants, and artificial tears. The presence of a watery, itchy, and red eye after an upper respiratory infection indicates a likely viral conjunctivitis, most likely caused by adenovirus. Incorrect Answers: B. Herpes zoster ophthalmicus occurs when the varicella zoster virus reactivates in the ophthalmic division of the trigeminal nerve. Most patients with herpes zoster ophthalmicus present with a periorbital vesicular rash distributed according to the affected dermatome. Poor nutrition, lack of sleep, and stress are aggravators that may activate the disease. Patients typically present with influenza-like prodromal symptoms for a week leading up to the disease, and pain and tingling in the dermatomal distribution occur before the appearance of the dermatomal rash itself. The rash presents with vesicles, and patients complain of unilateral eye pain and watery discharge. Treatment is with acyclovir. The patient's lack of a dermatomal, vesicular rash makes this answer choice unlikely. C. Staphylococcus aureus is the most common cause of bacterial conjunctivitis. Like viral conjunctivitis, preservation of visual acuity and pupillary reaction occurs. Unlike viral conjunctivitis, patients present with stinging pain and bilateral mucopurulent discharge, associated with "glued" eyes upon waking due to the mucopurulent secretions. Treatment is with azithromycin or ciprofloxacin ointment. The patient's discharge is watery, not mucopurulent, making this answer choice unlikely. D. Chlamydial conjunctivitis should be suspected in adults engaged in sexual activity who do not improve their symptoms with antibiotic treatment. Like bacterial conjunctivitis, patients' visual acuity and pupillary reaction are typically preserved. There is also a mucopurulent discharge associated with "glued" eyes upon waking. The key difference is the history of STDs and ongoing sexual activity that does not get better with standard treatment for bacterial conjunctivitis. Treatment is with topical erythromycin plus oral

A geriatric client is complaining of difficulty hearing. He relates that the problem seems to have gotten worse over the past few years. What finding would support a diagnosis of presbycusis? A. Complain that they can hear voices, but everyone mumbles B. Rinne test air conduction greater than bone conduction C. History of long term use of tetracycline antibiotics for chronic infections D. Family history of chronic hearing loss

Correct Answer: A. Complain that they can hear voices, but everyone mumbles In presbycusis, the ability to hear high frequency sounds is diminished. Patients with presbycusis have difficulty distinguishing consonant sounds, so words such as shoe and true are heard as "oo". Incorrect Answers: B. The Rinne test is normal if air conduction > bone conduction. An abnormal Rinne test, with bone>air conduction, is consistent with conductive loss, particularly if the Weber test also lateralizes to that side.However, these classifications are not useful in diagnosis of presbycusis as it is multifactorial and the pathophysiology is complex. C. D. Tetracyclines are not ototoxic, and family history may or may not contribute to the problem. Vital Concepts: In presbycusis, the ability to hear high frequency sounds is diminished. Patients with presbycusis have difficulty distinguishing consonant sounds, so words such as shoe and true are heard as "oo". References: Zerwekh, JoAnn G, and Jo C. Claborn. Adult Nurse Practitioner Certification Review. Philadelphia: Saunders, 2004. Fitzgerald. Nurse practitioner certification examination and practice preparation. 2010. Maria T. Codina Leik. Adult-Gerontology Nurse Practitioner Certification Intensive Review: Fast Facts and Practice Questions, Second Edition Paperback - August 12, 2013

A 7-year-old is found to have a white mass behind the tympanic membrane on the left side during routine physical exam. Audiogram confirms conductive hearing loss in the left ear. He has no history of previous ear surgery, and you do not appreciate any retraction of the tympanic membrane. What is the diagnosis? A. Congenital cholesteatoma B. Primary acquired cholesteatoma C. Secondary acquired cholesteatoma D. Glomus tympanicum

Correct Answer: A. Congenital cholesteatoma A white mass behind the tympanic membrane in an otherwise asymptomatic child is most consistent with a congenital cholesteatoma. The exact etiology remains controversial, but if it grows to a large enough size, it can lead to a conductive hearing loss. Incorrect Answers: B. Primary acquired cholesteatoma develops because of negative middle-ear pressure, which causes a retraction of the tympanic membrane, most frequently in the pars flaccida. C. Secondary acquired cholesteatoma develops because of epithelial cells being introduced into the middle ear such as with pressure equalization tube placement or trauma. D. Glomus tympanicum is a paraganglioma (neuroendocrine tumor) that develops in the middle ear. It presents as a reddish bulge behind the tympanic membrane and not a white mass.

A 25-year-old runner with no PMH complains of pain in the ball of her foot. She has been experiencing this pain for 3 months, and the pain also spreads to her toes occasionally. It is mitigated by rest and relaxation and is aggravated by running. The pain occurs at all times of the day and is not related to psychological stress, sleep, or temperature. She recently started running a lot more every day and has noticed that her new shoes seem a little tight. The patient's temperature = 98.3°F, HR = 70, RR = 16, and BP = 115/75. Physical exam reveals tenderness over the ball of the foot and over her metatarsals as well. Her foot also appears to be quite arched. The patient has no trouble putting weight on the foot and does not have any erythema, swelling, or discoloration. What is the best step in management? A. Conservative therapy (rest, ice, NSAIDs) B. X-ray of foot C. MRI of foot D. Arch support and orthotics of foot

Correct Answer: A. Conservative therapy (rest, ice, NSAIDs) Metatarsalgia is pain on metatarsal heads that results in tenderness over the ball of the foot and sometimes the toes. Patients who run a lot or who wear tight-fitting shoes often complain of these symptoms. The symptoms are worsened by running and improve with rest and relaxation. It also presents with an arched foot and excess pronation of the foot. Metatarsalgia is not a serious condition and does not result in complications typically. Most cases of metatarsalgia heal within months, and this self-limited condition can be treated with conservative management. Incorrect Answers: B. and C. Stress fractures are fatigued-induced fractures of the bones of the body from repeated overuse. They are diagnosed by X-ray and MRI of the foot. Rather than occurring suddenly, the fracture happens gradually, especially in athletes such as long-distance runners. Patients describe initially dull pain that increases in severity and frequency over time, sometimes to the point that the pain may occur at rest. Physical exam reveals a lack of crepitus, possible deformation, and sometimes leg-length discrepancy. Our patient does not exhibit signs of a stress fracture, stress fracture typically does not present with the ball of the foot pain, and there is no worsening of the pain over time. Thus, no imaging is indicated here. D. Arch supports and orthotics are a reasonable way to treat metatarsalgia. However, it is easier to start with conservative management first, and if that fails, then this is a viable option.

A 35-year-old pregnant G1P0 at 10 weeks of gestation presents for a regular check-up. She is Rh(+). Titers reveal that she is not immune to rubella. During the course of her pregnancy, which of the following vaccinations should this patient receive? A. Flu and Tdap B. Rho(D) immunoglobulin, flu, and Tdap C. Flu and MMR D. Tdap and varicella

Correct Answer: A. Flu and Tdap Inactivated flu vaccine is recommended for all persons >6 months of age annually. Since the vaccine contains inactivated viral components, it is safe and highly recommended during pregnancy. Tdap is also recommended for all pregnant women, regardless of when their last Tdap/Td booster was administered. Tdap is also safe since it is inactivated and optimally given between 27 and 36 weeks of gestation. In general, avoid any live vaccines in pregnant women. These include HPV, MMR, varicella, and live attenuated flu vaccine (intranasal). Incorrect Answers: B. While both the flu and Tdap vaccines are indicated in this pregnant patient as discussed above, Rho(D) immunoglobulin is not. The latter should be given to Rh(-) pregnant women with negative Rh titers to protect against the development of Rh antibodies in case the fetus is Rh(+). This patient is Rh(+), and therefore, will not develop anti-Rh antibodies even if the fetus is Rh(+). Therefore, Rho(D) immunoglobulin is not indicated. C. The inactivated flu vaccine should be administered to all pregnant women. However, MMR is a live- attenuated vaccine and should not be given to pregnant women or severely immunocompromised patients (such as HIV+ patients with CD4+ counts <200) since there is a risk of infecting the fetus. This patient is rubella non-immune and should receive an MMR vaccine postpartum but not during pregnancy. D. Tdap is indicated in all pregnancies. However, varicella is a live virus and should be avoided in pregnancy.

A 14-year-old female presents for a pre-sports physical. She complains of back discomfort when she "sits at school for a long time." Her exam reveals scoliosis with a left-sided curve. An MRI of the spine is indicated to rule out which of the following conditions? A. Intraspinal syrinx B. Narrowing of disk space C. Schmorl nodes D. Irregularities in vertebral endplates

Correct Answer: A. Intraspinal syrinx About 80% of scoliosis cases appear as a right thoracic curve. A left-sided pattern is associated with risk for intraspinal syrinx or tumor, which can be detected on MRI. The remainder of findings are associated with Schneurmann kyphosis, the 2nd-most common cause of spinal deformities in pediatrics. A scoliotic curve must measure greater than or equal to 10 degrees on a spinal radiograph via the Cobb method (a special tool that measures the angle on radiograph) to meet the criteria for diagnosing scoliosis, but most patients do not exhibit clinically significant respiratory symptoms until the curves measure 60 to 100 degrees. The USPSTF states that treating idiopathic scoliosis during adolescence leads to health benefits in only a small fraction of patients. Most of the cases detected through screening do not become clinically significant, and scoliosis that requires aggressive treatment/operative treatment is likely to be detected without screening because these patients are eventually symptomatic. Incorrect Answers: (B) Narrowing of disk space can be evaluated with an x-ray. Given this patient's left-sided curve, the intraspinal syrinx is more important to rule out with an MRI. (C) Schmorl nodes are intervertebral disc herniations that cause protrusion into another vertebra. While this may cause the patient's back discomfort, it would not lead to a left-sided pattern which was found on the physical examination. (D) Irregularities in vertebral endplates are associated with degenerative disc disease.While this may cause the patient's back discomfort, it would not lead to a left-sided pattern which was found on the physical examination.

The NP sees a patient with left-sided facial paralysis. The patient cannot completely close her left eye and her smile is asymmetrical. Following a thorough neurological exam, the NP determines that the patient has paralysis of cranial nerve VII and makes the determination that the patient likely has Bell's Palsy. Laboratory testing in patient's with Bell's Palsy, (when indicated) includes which of the following? A. Lyme disease antibody titer and/or blood glucose B. CBC C. CT scan of the head and neck D. BUN and creatinine

Correct Answer: A. Lyme disease antibody titer and/or blood glucose Bell's Palsy is a peripheral palsy of the facial nerve which causes muscle weakness on one side of the patient's face. If the NP does not suspect other brain dysfunction, the Lyme disease antibody titer is the most appropriate test to order if there is an appropriate history, as Bell's Palsy may be a complication of Lyme disease. In addition, there is an association between Bell's Palsy and Diabetes and a blood glucose is warranted if the patient has the appropriate risk factors. Another possible etiology is herpes simplex virus type I. Most cases of Bell's Palsy will resolve spontaneously without treatment (70-80 %), though therapy with Acyclovir and steroids has the potential to shorten the duration of the disease. Incorrect Answers: B. Although the NP is likely to order a CBC, this is not the most appropriate next diagnostic test. C. If the NP does not suspect other neurological problems, a CT scan of the head and neck is unlikely to provide diagnostic information. D. The BUN and creatinine are unlikely to provide valuable diagnostic information.

A 6-year-old boy arrives at the office with his parents for a wellness check. What are the three most common types of fractures in children? A. Metaphyseal, epiphyseal, apophyseal B. Comminuted, epiphyseal, spiral C. Transverse, metaphyseal, apophyseal D. Comminuted, spiral, transverse

Correct Answer: A. Metaphyseal, epiphyseal, apophyseal Children are different from adults in that they have a metaphysis and a physis which is also known as the growth plate. This is the area of bone where it lengthens. Salter-Harris fractures are epiphyseal-metaphyseal fractures. Children also have apophyses in many locations in the body, including the heel (Sever's disease) and knee (Osgood-Schlatter's disease). Incorrect Answers: B. A comminuted fracture is a fracture that involves more than 2 bone components. A spiral fracture is a torsion fracture; when observed in long bones of young children, child abuse must be considered. C. A transverse fracture can occur in children, but epiphysis fractures are more common. A transverse fracture is perpendicular to the axis of the bone. D. These are all types of fractures that occur in children, but they are less common.

Which condition is associated with wearing ill-fitting, high heeled, or pointed-toed shoes? A. Morton's neuroma B. Plantar fasciitis C. Cauda equina syndrome D. Hallus rigidus

Correct Answer: A. Morton's neuroma Morton's neuroma is fibrosis (neuroma) is caused by compression and trauma to the interdigital plantar nerve, usually caused by ill-fitting shoes (heels or pointed toes). Patients may complain of pain and burning in third web space and may have palpable "pebble-like" nodule between third and fourth toe. Incorrect Answers: B. Plantar fasciitis is associated with pain on the bottom of the foot, including the arch and heel that is worse with weight bearing. Pain may be worst in early morning. Disorder associated with overuse, poor arch support, and trauma to fascia. C. Cauda equina is associated with spinal stenosis, bulging disks or other conditions that affect the nerve roots of the lumbar spinal cord. It presents as saddle numbness and may also have incontinence of bladder and bowel. This condition needs immediate ED referral. D. Hallus rigidus is inflexibility of the great toe, usually caused by joint changes. It presents with decreased ROM of joint and pain with ambulation.

A 19-year-old male is evaluated at a college health clinic for a 2-week history of cough and rhinitis. He complains of fatigue. His cough is productive of small amounts of clear sputum. The patient has no significant past medical history and takes no medication. His temperature is 100 degrees Fahrenheit, his heart rate is 100/minute, and his respiratory rate is 20/minute. Examination reveals fine crackles in the right lower lobe of the lung. Chest X-ray reveals diffuse infiltrates in the right lower lung field. Which is the most likely diagnosis? A. Mycoplasma pneumoniae B. Legionnaire's disease C. Asthma D. Streptococcal pneumonia

Correct Answer: A. Mycoplasma pneumoniae Mycoplasma pneumoniae is an atypical bacteria that causes "walking pneumonia," a highly contagious respiratory infection that is most common in children and young adults. The classic presentation is a young adult complaining of several weeks of fatigue accompanied by severe paroxysms of coughing with scant sputum production. Onset of symptoms may be gradual. Patients frequently complain of a cold, clear rhinitis, or a sore throat with low-grade fever. Wheezing and diffuse crackles are heard on auscultation of the lungs. The nose may have clear mucus. The throat may be erythematous without pus or exudate. Chest radiograph usually reveals diffuse interstitial infiltrates, with a pleural effusion in up to 20% of patients. CBC may be normal. Incorrect Answers: B. Legionnaire's disease refers to pneumonia caused by legionella pneumophilis, an important cause of nosocomial and community-acquired pneumonia. It is often accompanied by acute diarrheal illness. C. Asthma is not characterized by an infiltrate on chest X-ray. D. Streptococcal pneumonia would result in a lobar infiltrate on a chest X-ray. It is also accompanied by a high fever and chills, productive cough, with large amounts of green to rust-colored sputum.

A 2-week-old infant presents for a newborn exam. He was born via spontaneous vaginal delivery at 38 weeks without any complications. The infant has an an elevated phenylalanine level. The mother tells you that she exclusively breastfeeds. No abnormalities are noted on physical examination. What is the next step in management? A. Order plasma amino acids B. Order assay for dihydropteridine reductase C. Switch to low-phenylalanine formula D. Switch to soy formula

Correct Answer: A. Order plasma amino acids A plasma amino acid level should be obtained to confirm the suspected diagnosis. Incorrect Answers: B. Once a diagnosis of phenylketonuria (PKU) is confirmed via plasma amino acids, the type of PKU can be assessed. PKU is either caused by a deficiency of phenylalanine hydroxylase (more common) or defective biosynthesis of tetrahydrobiopterin, a cofactor required for phenylalanine hydroxylase to function. C. Dietary treatment with a low-phenylalanine formula should not be started prior to obtaining a confirmed diagnosis. D. Soy formula is required for patients with galactosemia and not phenylketonuria.

What is a frequent cause of exercise-induced patellar pain and swelling at the tibial tubercle in pre-adolescent girls and early adolescent boys? A. Osgood-Schlatter disease B. Charcot-Marie-Tooth disease C. Baker's cyst D. Osteochondritis dissecans

Correct Answer: A. Osgood-Schlatter disease Osgood-Schlatter disease refers to a microfracture at the ossification center near the tibial tubercle, where the patellar tendon inserts. Incorrect Answers: B. Charcot-Marie-Tooth disease is characterized by a cavus foot and familial neuropathy. C. Baker's cyst results from stretching of the gastrocnemius and semimembranosus bursa. It is usually painless during childhood. D. Osteochondritis dissecans results from a vascular insult to the articular cartilage at the knee, most commonly at the lateral aspect of the medial femoral condyle.

An 11-year-old boy was tackled while playing football and landed on his shoulder. He immediately complained of upper chest pain and dyspnea. On exam, he has mild neck-vein engorgement and a swollen sternoclavicular joint. What is his most likely diagnosis? A. Posterior sternoclavicular joint dislocation B. Clavicle fracture C. Cervical strain D. Rib fracture

Correct Answer: A. Posterior sternoclavicular joint dislocation Posterior sternoclavicular joint dislocations are rarer than anterior sternoclavicular dislocations but are a life- threatening emergency when they do occur. Open or closed reduction should occur as soon as possible while the patient is monitored and under controlled conditions in the presence of a cardiothoracic surgeon. Patients with sternoclavicular (SC) injury usually have anterior chest and shoulder pain that is worsened by arm movement. Patients with posterior SC dislocation may also complain of difficulty breathing, dysphagia, or upper extremity paresthesias. If the posterior displacement of the medial clavicle or clavicular fracture injures the recurrent laryngeal nerve, hoarseness may also be present. Displacement may be difficult to appreciate. Incorrect Answers: B. With his physical exam findings, clavicle fracture is less likely than posterior SC joint dislocation. Clavicle fracture involves pain on palpation and typically deformity of the clavicle. C. Cervical strain presents as neck pain, often after an acute injury like whiplash during a motor vehicle accident. A strain can sometimes cause paresthesias if there is a muscle spasm that irritates a nerve running through it. It is not associated with symptoms like the case above. D. A rib fracture will cause pain with respiration but is unlikely to cause the symptoms listed above. The mechanism of injury also make it less likely.

A 17-year-old girl who fell on her right arm 8 hours ago on a remote hiking trail is evaluated in the emergency department. Her elbow is swollen and painful, and she is hesitant to move it. She has a loss of capillary refill in her right fingers, without sensation. An X-ray shows a fracture-dislocation of the humerus at the elbow. What is a contraindication to open reduction and internal fixation (ORIF)? A. Prolonged ischemic time B. Severe contamination C. Advanced age D. Potential for further damage E. Long length of immobilization

Correct Answer: A. Prolonged ischemic time Prolonged ischemia (>6 hours) makes surgical treatment short of amputation futile. Other contraindications to ORIF include active infection or osteomyelitis, poor soft-tissue quality overlying the injury, medical conditions that contraindicate surgery or anesthesia (recent myocardial infarction), and cases in which amputation is a better solution than attempted reduction and fracture fixation. Incorrect Answers: B. Areas of severe contamination must be decontaminated prior to final stabilization. C. Advanced age is not a contraindication. D. Reducing the potential for further damage is an indication for surgery. E. With advanced surgical techniques, the period of immobilization is reduced and no longer poses the same problem.

A 31⁄2-year-old boy with Down syndrome presents to his otolaryngologist for routine evaluation. The patient's mother reports that he is doing well. She denies snoring or apneic episodes at night. She also denies changes in his gait or use of his arms and hands, neck pain, or torticollis. His most recent hearing test, performed 6 months ago, was normal. TMs are poorly visualized due to canal stenosis. Which of the following is most appropriate? A. Recommend polysomnogram B. Follow-up in 1 year C. Order lateral neck plain film D. No further hearing testing needed

Correct Answer: A. Recommend polysomnogram According to the National Down Syndrome Society, nearly 60% of children with Down syndrome have obstructive sleep apnea by the age of 31⁄2 to 4 years. This is attributable to a multitude of factors, including low muscle tone of the mouth and pharynx, narrow air passages, a large tongue, and adenotonsillar hypertrophy. Recognizing the poor correlation between patient report and polysomnogram results, the American Academy of Pediatrics recommends that all children with Down syndrome undergo a formal sleep study by the age of 4 years. Interval examinations should be performed every 3-6 months until the TM can be visualized and reliable tympanometry can be performed. Incorrect Answers: B. As the TMs in this patient are poorly visualized, 1-year follow-up is inappropriate. C. There is an associated risk of atlantoaxial instability in Down syndrome children, but routine evaluation of the cervical spine in asympomatic children is not recommended. D. Routine hearing testing is performed every 6 months until ear-specific normal hearing levels are established (usually at age 4) and then annually thereafter.

A 45-year-old female presents with a known history of depression. She reports taking Escitalopram with positive relief. Which of the following statements is false regarding SSRIs? A. SSRI-induced sexual dysfunction occurs only in men. B. SSRI-induced sexual dysfunction affects both libido and orgasms. C. Weight gain may occur more frequently with paroxetine. D. Weight gain, fatigue, and apathy are infrequent side effects.

Correct Answer: A. SSRI-induced sexual dysfunction occurs only in men. SSRI-induced sexual dysfunction occurs in both women and men. This generally fails to diminish with ongoing treatment, but it does dissipate on discontinuing treatment. Incorrect Answers: (B), (C), (D) are all true regarding SSRIs.

A 56-year-old female presents to the allergist with a stuffy nose. She has been taking the topical nasal decongestant Afrin for the past month and is unable to stop taking it without having great trouble breathing through the nose. She is diagnosed with rhinitis medicamentosa. Which of the following could be used to wean her from this topical decongestant? A. Systemic steroids B. Systemic antibiotics C. Topical decongestants D. Systemic antihistamines

Correct Answer: A. Systemic steroids Systemic steroids and systemic decongestants have been shown to be effective in weaning a patient from topical decongestants. The nasal mucosa in patients with allergic rhinitis usually looks edematous and pale, whereas the nasal mucosa in patients with non-allergic rhinitis is usually normal in color and edematous red in patients with acute viral rhinosinusitis or rhinitis medicamentosa. Complete recovery of the nasal mucosa can take as long as 1 year for long-term users. Incorrect Answers: B. Systemic antibiotics are not indicated in the treatment of rhinitis medicamentosa. C. Topical decongestants are the cause, not the treatment for, rhinitis medicamentosa. D. Systemic antihistamines have no efficacy in rhinitis medicamentosa since this is not associated with degranulation of mast cells and no histamine is released Vital Concept: Topical decongestants are the cause of rhinitis medicamentosa and should only be used on a time-limited, short term basis.

The nurse practitioner is examining lymph nodes in the neck. What is palpated in the anterior triangle of the neck? A. Posterior cervical chain B. Anterior superficial chain C. Periauricular lymph nodes D. Supraclavicular lymph nodes

Correct Answer: B. Anterior superficial chain The sternocleidomastoid muscle is the division between the anterior (containing the anterior cervical chain) and posterior triangles. Incorrect Answers: A. The trapezius muscle marks the posterior border of the posterior (containing the posterior cervical chain) triangle. C. The Periauricular lymph nodes are not palpated in this area. D. The supraclavicular or scale nodes are palpated in the angle formed by the clavicle and the sternocleidomastoid muscle. Vital Concepts: The sternocleidomastoid muscle is the division between the anterior (containing the anterior cervical chain) and posterior triangles.

A 28-year-old female presents to your clinic with a sore throat that began the night before. She denies any past medical history. She did not feel well for about a day, and shortly after dinner last night, her throat began to hurt, and swallowing was very painful. Upon arrival, her vitals are blood pressure (BP) 110/80, heart rate (HR) 110, respiratory rate (RR) 18, Temp 101.0, and oxygen saturation 99% on room air. It sounds like she has food in her mouth, and she is drooling a little bit. She has no adenopathy, and you can barely see her erythematous pharynx due to pain and gagging. What is recommended for this patient? A. Presumptively treat for streptococcal infection with course of amoxicillin B. Call 911 and send to emergency room C. Draw a complete blood count with blood cultures and treat presumptively for staphylococcal infection D. Get chest and lateral soft tissue neck X-ray; if not normal, treat with amoxicillin

Correct Answer: B. Call 911 and send to emergency room This patient has a "hot potato" muffled voice consistent with epiglottitis or another acute upper-airway obstruction. The rapid-onset sore throat, fever, and drooling all support the diagnosis of possible epiglottitis. 50-80% of adult patients with epiglottitis present with this "hot potato" muffled voice. 30% also have stridor. Peritonsillar abscesses can also cause these symptoms and should be on your differential as well. The patient needs to be sent to the ER immediately to stabilize her airway if necessary with intubation. In less obvious cases, perform a lateral soft tissue neck XR, but in this case, do not waste the time doing so. Incorrect Answers: (A) This patient is actively having respiratory distress that requires immediate care. (C) This patient is actively having respiratory distress that requires immediate care. (D) This patient is actively having respiratory distress that requires immediate care. Vital Concepts: This patient has a "hot potato" muffled voice consistent with epiglottitis or another acute upper-airway obstruction. The rapid-onset sore throat, fever, and drooling all support the diagnosis of possible epiglottitis. Peritonsillar abscesses can also cause these symptoms and should be on your differential as well. The patient needs to be sent to the ER immediately to stabilize her airway if necessary with intubation.

A one week-old newborn is noted to have bilateral bloody eye discharge. The baby was born to a 21-year-old G1P1 mother who had no prenatal care. On exam he has red and thickened conjunctivae. What is the most likely cause of the neonate's eye problems? A. Silver nitrate B. Chlamydial conjunctivitis C. Gonococcal ophthalmia D. S. aureus

Correct Answer: B. Chlamydial conjunctivitis Since the mother received no prenatal care, she could have had an STD and not known. Chlamydial conjunctivitis is a primary cause of ophthalmia neonaturm and usually occurs 5-25 days after birth. The conjunctiva may become friable, causing bloody discharge. Diagnosis is made by a nucleic acid amplification test (NAAT). Incorrect Answers: A. Silver nitrate irritation occurs within the first day of life. C. Gonococcal conjunctivitis is also a possibility for this patient, but it usually occurs within the first few days of life. D. S. aureus is less common than chlamydial conjunctivitis and would not cause bloody discharge. References:

An 11-year-old male was playing baseball as a pitcher when he took a line drive to the chest. He immediately collapsed on the field and was unresponsive. What is his most likely diagnosis? A. Mediastinal carcinoma B. Commotio cordis C. Rib fracture D. Pulmonary contusion

Correct Answer: B. Commotio cordis Commotio cordis is a cause of sudden cardiac death in sports. Common sports are baseball, lacrosse, and hockey. Softer balls in have been attempted in baseball, and rule changes are also thought to decrease risk, but evidence has not yet shown this to be true. Incorrect Answers: A. Mediastinal carcinoma may impinge on vital structures but does not often lead to a sudden collapse. C. Rib fractures usually result from a major trauma like a motor vehicle accident, even if a rib had a pathologic weak point from a metastasis. D. Pulmonary contusion is possible but less likely to lead to immediate collapse.

Which structure of the eye is responsible for color vision? A. Macula B. Cones C. Rods D. Sclera

Correct Answer: B. Cones Cones are responsible for color vision and work best in bright light. Incorrect Answers: A. The macula helps you see fine details from the center of your vision.C. Rods are responsible for night vision.D. The sclera is the protective covering of the eye. It is not responsible for vision


Set pelajaran terkait

Lesson 4: Comparing Local Networking Hardware

View Set

MARK3336 Ch. 5 Video: IMG World: Creating the College Experience

View Set

Med/Surg 3 Chapter 11 NCLEX Style Questions

View Set

Ch 05 - Infertility, Contraception, & Abortion

View Set

CHAPTER 6: "The debate over the Alien and Sedition Acts of 1798 revealed bitter controversies on a number of issues." Discuss the issues involved and explain why these controversies developed."

View Set

Chapter 26: The Triumph of Conservatism (HHAS Exam)

View Set